Está en la página 1de 49

238

Academy

GENERAL ENGLISH

ACE

GENERAL ENGLISH
COMPREHENSION
Directions
In this sections you are given short passages . After each passage , you will find a number of
questions based on the passage. First, read the passage and then answer the questions based
on it . You are required to select your answers based on the contents of the passage and
opinion of the author only.
Examples I and J are solved fir you .

2011
PASSAGE I
If we think about the men an women whose judgment on practical affairs and on conduct we
respect, we should certainly agree that academic education did not seem to be very important
in their being. We should say that some of them were learned and others not, some rich,
others poor. They had no special training or accomplishment. That is why we contrast the
one-sidedness of the expert with the good sense or common sense of an ordinary man and
why democrats think that the proposals of the expert should be improved by the ordinary
man.
01. The use of the epithet academic is
(a) honorific
(b) referential
(c) pejorative
(d) sardonic
02. Sound judgment derives from
(a) meditation
(b) emotionality

(c) specialization

(d) experience

03. The observation that men and women of (sound) judgment have no special training is
(a) a sweeping generalization
(b) indifferent
(c) condensed
(d) superimposed
04. The inference drawn by the author in this passage is
(a) balanced
(b) self-contradictory (c) open-ended

1.b

2. d

3. a

(d) ambiguous

4. c
PASSAGE II

The clock struck a quarter past nine as Mary hurried into the big block of offices where she
was going to work. Her bus had crawled through the dense city traffic, making her a few
minutes late for her very first job. She resolved to leave home earlier the next day.
Inside the building she had to wait once again, this time for the lift to take her up to the sixth
floor. When at last she reached the door marked J. King, Manager, she knocked rather
nervously and waited. There was no answer. She tapped on the door again, but still there
was no replay. From inside the next room she could hear the sound of voices, so she opened
the door and went in.

239
Academy

GENERAL ENGLISH

ACE

05. May was late for the office because


(a) it took time for her to walk through the dense city traffic
(b) She had to cross several blocks of offices to reach her office buildings
(c) She started late from home
(d) the bus took more time for the journey than expected
06. Mary was nervous because
(a) She was afraid of seeing the Manager
(b) She was new to the city
(c) She was late
(d) of the strain of the journey to office
07. Consider the following statements:
Mary opened the door and entered the room because
1. She was impatient and impolite
2. She was nervous
3. She though that the Manager hadnt come although other office workers have
Which of the statements given above is/are correct?
(a) 1 only
(b) 2 and 3 only
(c) 3 only
(d) 1,2 and 3
08. The impression Mary gave is of
(a) an unpunctual person
(b) a person who cannot manage her own affairs
(c) a person who had not worked in any office before
(d) a restless person

5. d

6. a

7. b

8. c

2010
PASSAGE-I
On the day of the Christmas concert the entire orchestra entered the gymnasium where a
large throng of pupils and parents were gathered. The school choir had gathered on the
balcony with candles and the orchestra began Silent Night. Slowly the lights dimmed,
leaving us in darkness except for the hundred glimmering candles held by the choir as they
softly sang the carol. Slowly, and just as dramatically, the orchestra began to falter-in direct
proportion to the dimming of the lights, when the lights faded out completely, so did the
orchestra, because no one could read his music. Except for me, for my music was in my head.
From a corner of the stage, the sound of my violin filled the gymnasium. The choir and I
went through verse after verse. When it was all over, the ovation was positively thunderous.
01. The narrator is
a). priest
b). the leader of pupils

c). an instrumentalist
d). a singer

ACE Academy
240

COMPREHENSION

02. W0hy did the orchestra begin to


falter?
a). The orchestra was not led

c) The violin dominated the other


instruments.

properly

b). The dimming of lights caused it


03. How could the choir sing when
the lights dimmed?

c).The audience made circles and


danced in ecstasy.
d). The audience made a noise
resembling thunder.

a).They had memorized the songs.


b). They could sing in candle light.

05. How was it possible for the narrator


to continue with his music?

c).There was some natural light at


the choir side.

a). He was at the corner of the stage


where he could get light.

d). The narrator helped them sing.


04. The statements that the ovation was
positively thunderous means that
a).There was a joyful expression of
appreciation by people

b) His instrument did not nee


power.

electric

c). He played in candle light.


d). He did not need to read his
music.

b). The audience joined the singers


and sang aloud.
Key for 2010 paper
1.c
2.b
3.d
4.a

d). There was no coordination in the


chorus.

5.d

PASSAGE-II
Patriotism is easy enough during war. For one thing, people become actively conscious of
their country when it is threatened. For another, since everyone or nearly everyone feels it,
there is a strong collective emotion flowing through the country. But when the war is over,
the country recedes from the consciousness and it is rarely even at the back of the mind. The
question, Am I doing any good to the country? occurs only rarely, A limited patriotism is
better than none, but it is not enough.
01. The author says that one reason why
patriotism is easy during war-time is
that
a). war is the only factor responsible
for arousing patriotism.
b). When the country is in danger, we
become conscious of it.
c). We become patriotic when we are
in a fighting mood.
d). Patriotism is always in the
consciousness.

02. Another reason suggested by the


author for the emergence of patriotic
feeling during war-time is that
a). patriotism is specially required
during war-time
b). we become deeply conscious of
danger to ourselves during war-time
c). Patriotism becomes contagious
because one finds a lot of other
people feeling patriotism

241
Academy

GENERAL ENGLISH

d). Every one feels that he/she should


strive to protect the others.

c). People are not capable of understanding


what is good and desirable for them.
d). The moral values which they have so
far upheld would disappear from their
minds.

03. When the threat of war is over,


a). there is no need for people to be
interested in others welfare.
b). People tend to become totally
unmindful of national interests.
c). people tend to become totally
unmindful of national interests.
d). People are generally anxious to
serve the country.

05. By a limited patriotism, the author


refers to
a). the peoples attitude of indifference to
the welfare of their country
b). the attitude of people who love other
countries as much as they love their
own country.

04. When the author says that the country


recedes from the consciousness, he
means to say that

c). Those who are self-centered and mainly


concerned about their own welfare
d). The attitude of people who are
indifferent to the welfare of their
country except during war-time

a). people no longer feel that they


belong to the country.
b). People do not have the country clearly
in the forefront of their mind.
Key for 2010 paper
1.b
2.c
3.c
4.b

ACE

5.d

PASSAGE-III
Most people lead the lives that circumstances have thrust upon them. But Wilson had
boldly taken the course of his life into his own hands. At 35, he had quit his job to lead a
pleasant life on an exotic island with just enough money to last for twenty five years. Once,
fifteen years after he had been on the island, I happened to meet him and enquired about his
financial situation. He said, It will carry me on till I am sixty. But one cannot be sure of
dying at sixty, I said.
1. According to the author, most people

b). Saved a lot of money to lead a


pleasant life.
c). Preferred to live in isolation.

a). do not allow circumstances to affect


their lives.
b). Have fatalistic attitude towards life.
c). Do not know how to cope with their
situation.
d). Do nothing to change the condition
they live in.

d). Wanted to live without depending


on others.
3. ..depends on oneself suggests
that Wilson, at 60, would
a).depend on his own resources
b). surrender himself to destiny
c). take way his own life.
d). Carry on living in the same way

2. Wilsons boldest decision was that he a).


quit his job.
Key for 2010 paper : 1.b

2.c

3.c

PASSAGE-IV

ACE Academy
242

COMPREHENSION

A soap opera is a kind of a sandwich, whose recipe is simple enough. Between thick slices
of advertising, spread twenty minutes of dialogue, and predicament, villainy, and female
suffering in equal measure, throw in a dash of nobility, sprinkle with tears, season with organ
music, cover with a rich announcer sauce, and serve five times a week. It may also contain a
teaser, a give away, a contest offer, or a cowcatcher or hitch-hike for another of the sponsors
products. It is the hope of every advertiser to habituate the housewife to an engrossing
narrative whose optimum length is forever and at the same time to saturate all levels of her
consciousness with the miracle of a given product, so that she will be aware of it all the days
of her life and mutter its name in her sleep.
1. The second sentence of the passage
contains the description of a/an
(a) programme announcement.
(b) recipe for sandwich.
(c) advertisement.
(d) melodrama.

(c) Satirical style.


(d) Sentimental and melodramatic
style.
3. According to the author, the
advertisers aim at
(a) educating and enlightening the
housewives.
(b) brainwashing the housewives.
(c) selling their wares very cheap.
(d) offering pure and wholesome
entertainment

2. Which of the following accurately


describes the authors style in the
passage?
(a) Scientific and objective style.
(b) Romantic style
Key for 2010 paper
1.d
2.c
3.b

PASSAGE-V
Most of you probably did not see Mohan at close quarters. He had amazing qualities.
One of these qualities was that he managed to draw out the good in another person. The other
person may have had plenty of evil in him. But he somehow spotted the good and laid
emphasis on the good. The result was that the poor man had to try to be good. He could not
help it. He would feel ashamed when he did something wrong.
1. The author assumes that most of us
(a) have not seen Mohan
(b) have not heard of Mohan.
(c) are not well-acquainted with
Mohans powers.
(d) have not observed Mohans house
closely.
2. One of Mohans greatest qualities was
that
(a) he could completely destroy the
evil in another man.
(b) he could discover the good in
another man.
(c) he always maintained a good life.
(d) he always kept away from bad
men.

3. The other man is called poor because


(a) he usually came from a poor
family.
(b) he was always very evil.
(c) he must try to become good and
give up his evil ways.
(d) he did not know what to do and
was helpless.
4. The other man tried to be good because
(a) Mohan forced him to do so.
(b) he wanted to imitate Mohan.
(c) he was ashamed of doing a wrong
thing.
(d) he did not want to remain poor.

243
Academy

GENERAL ENGLISH

Key for 2010 paper : 1.c

2.b

4.c
Key: 1.c

4.c

2.b

3.c

ACE

3.c

2009
PASSAGE-1
An astrologer passing through the village called Koopal foretold that Velan would live in
a three-storeyed house surrounded by many acres of garden. At this everybody gathered
round Velan and made fun of him. For Koopal did not have a more ragged and
God- forsaken family than Velans. His father had mortgaged every bit of property he had and
worked with his whole family on other peoplelands in return for a few annas a week. A
three-storeyed house for Velan indeed!.... But those who made fun of him would have
congratulated the astrologer if they had seen Velan about thirty or forty years later. He
became the sole occupant of Kumar Baugh- that palatial house on the outskirts of Malgudi
town.
01. The astrologer
(a) visited the village
(b) belonged to the village
(c) happened to pass through the village (d) was invited by the villagers
02. The village where Velan lived was
(a) Malgudi
(b) Koopal
(c) Kumar Baugh

(d) None of the above

03. Everyone made fun of the forecast because


(a) Velan was too small to be the owner of the three-storeyed building
(b) Velan was a funny character
(c) In the present situation it was unbelievable
(d) his father had mortgaged the house
04. The villagers would have congratulated the astrologer because
(a) he indeed became the owner of the palatial house
(b) he could change the fate of Velan
(c) his forecast turned out to be a reality
(d) he was a man of great knowledge
PASSAGE-1I
I had undertaken this perilous swim, not to gain fame or trophies but to prove to the
world that Indians are no longer a fraid. To the youth of India this triumph would
dramatically demonstrate that nothing is impossible for them. All they have to do is to
believe and persevere, and the goal will be theirs ! For it is my firm conviction that
unless we individually become adventurous and nationally restless and enterprising.
India will not be able to break the bonds of apathy and tradition, whether on the physical
or on the intellectual fronts. My Indian Ocean venture was a humble contribution
towards this end.
05. The passage suggests that the author is a
(a) politician
(b) writer
06. The author swam because he wanted
(a) to become famous
(c) the young Indians to swim like him

(c) teacher

(d) famous swimmer

(b) to win awards

ACE Academy
244

COMPREHENSION

(d) the world to know that the modern Indians are fearless
07. To become winners, the author says the Indian youth must become
(a) richer

(b) persevering

(c) restless

08. The expression perilous swim means


(a) painful
(b) useless swim

(d) emotional

(c) risky swim

(d) interesting swim

PASSAGE-1II
Let us laugh, says, W. Mathews, it is the cheapest luxury man enjoys. It stirs up
the blood, expands the chest, clears away the cobwebs from the brain and gives the whole
system a healthy treatment. So is it not nice to laugh a lot? It is said, Laughter is the
best medicine. For those who dislike medicine, sweet or bitter, a good joke that provokes
laughter is prescribed. It is nice to have a good laugh but a guffaw may sometime lock
ones jaws and so it is suggested that those who enjoy a loud guffaw go slow and subside
into a gurgle but the best thing is, as done in Honble courts before Honble judges, just
titter. And finally, I feel that I should smile, laugh heartily (without the predicament of
lockjaw) and be able to enjoy all jokes including ones directed at myself. But never
making a laughing stock of myself in the process. Let us remember the wise saying,
He is not laughed at that laughs at himself.
09. It is said that laughter is the best medicine because
(a) it is the cheapest luxury man enjoys
(b) it is available free of cost
(c) it is cheaper whereas medicines in the shops are costly
(d) it provides better treatment than costly medicines
10. The writer says he would never make a laughing stock of himself. It means he should not
(a) let others ridicule him
(b) show disrespect to Judges
(c) laugh, but simply smile at others
(d) let others laugh
11. The writer feels laughter is to be prescribed to those who
(a) are seriously ill
(b) dislike medicines
(c) cannot buy medicines
(d) do not need medicines
12. By quoting a wise saying in the last line the writer
(a) reiterates the point that laughter is the best medicine for us
(b) guides us about what we should do from being laughed at
(c) cautions us as to how we should conduct ourselves in the Honble Courts
(d) censures those who cannot laugh at themselves
PASSAGE-1V
I once saw a tigress stalking a month old kid. The ground was very open and the kid saw
the tigress while she was still some distance away and started bleating, whereon the tigress
gave up her stalk and walked straight up to. When the tigress had approached to within a few
yards, the kid went forward to meet her, and on reaching the tigress stretched out its neck and
put up its head to smell her. For the duration of a few heart beats the month-old kid and the
queen of the forest stood nose to nose, and then the queen turned and walked off in the
direction from which she had come.
13. Stalking in this context means
(a) running threateningly
(b) moving about in circles

245
Academy

GENERAL ENGLISH

(c) targeting a victim

ACE

(d) moving quietly without being seen or heard

14. The tigress walked straight up to the kid because


(a) running would frighten the kid
(b) stalking was useless now
(c) the tigress was patient
(d) she was sure of her victim
15. The kids behaviour indicates that
(a) it was familiar with the sight of tigress (b) it was attracted by the tigresss smell
(c) it took the tigress for a playmate
(d) it was not aware of the danger involved
16. The expression for the duration of a few heart beats suggests
(a) the few seconds the tigress and the kid spent smelling each other
(b) the fear of the kid
(c) the excitement of the tigress
(d) the anxiety of the watcher
PASSAGE-V
The door led to the back of a steep tier of narrow wooden benches rising from the
lecturers desk like a football stand. Behind the desk were three large blackboards
screwed to the walls, which were otherwise panelled with stained perpendicular planks.
The roof was lost in a criss-cross of thin iron girders through which half a dozen electric
globes were suspended to supplement the thin light that filtered through the windows
under the eaves.
17. The passage describes
(a) a stadium
(b) a lecture theatre

(c) an operation theatre

18. The word eaves means


(a) heavy curtains
(c) over-hanging part of a sloping roof

(b) electric globes


(d) the ceiling

(d) a cinema hall

19. The writer says The roof was lost because


(a) it was very dark
(b) the roof was covered with soot
(c) the roof was covered with a large number of iron beams
(d) there was a canopy
20. What is the appropriate description of the passage?
(a) Objective
(b) Rhetorical
(c) Impressionistic

(d) Verbose

Keys:
01.c 02.b

03.c

04.c

05.d

06.d

07.b

08.c

10.a

13.d

15.d

16.d

17.b

18.c

19.c

20.d

14.d

09.a

11.b

12.b

2008
J.K Galbraith has described the current inflation as a revolt of the rich against the poor.
Richard Parker supports this view with the claim that it is the richest people that benefit by
inflation: while the rest, especially the poorest, suffer in proportion to their relative property.
On the other hand, a 1979 study published by the Brookings Institute indicates that the lower
classes are generally benefited by inflation, while the upper classes lose. The confusion over
who suffers from inflation is extended to other questions such as the major causes of
inflation. In addition to the proponents of the demand theory, there are many who doubt
whether inflation is essentially an economic problem at all. Konard Kellen holds that inflation
is not an economic problem but a psychological one. Paul Samuelson cites a social factor, the

ACE Academy
246
evaluation

of

COMPREHENSION
more

humane

society,

as

root

cause

of

inflation.

01. What does the author believe?


(a) Only the rich suffer from inflation
(b) Only the poor suffer from inflation
(c) Both the rich and the poor suffer from inflation
(d) It is debatable as to who suffers from inflation
02. From the passage, which one of the following may be inferred?
(a) The effect of inflation can be studied with scientific precision
(b) The effect of inflation cannot be studied with scientific precision
(c) Economists are biased towards the rich or the poor
(d) The effect of inflation varies from place to place, and people to people
03. On the basis of the information given in the passage, what are the causes of inflation?
(a) Essentially economic
(b) Essentially psychological
(c) Essentially social
(d). Highly controversial
04. How does the author write?
(a) Objectively
(b) Critically

(c) Sympathetically

(d) Persuasively

PASSAGE II
Work itself is a very important factor in motivating a person. A person tends to perform a task
more enthusiastically if that work affords more satisfaction than the other work. A persons
satisfaction out of work is more is it is more need satisfying. A man seeks something from
work and if he gets more satisfaction from a particular work he will be prone to do that work
better or harder. There are incentives for work; these can be financial or non-financial. People
like missionaries and some scientists do not work basically for material gains as such. A
person
wants
to
do
a
work
which
is
personally
meaningful.
05. Which one of the following is correct?
(a) Work is the most important factor by which a person is motivated.
(b) Work is the least important factor that motivates a person
(c) Among the several factors that motivate a person, work is an important one.
(d) Nothing can motivate a person who does not have some permanent work.
06. Which one of the following is correct? A persons work satisfaction depends on
(a) the money and other benefits he receives from work
(b) the need satisfying nature of the work he does
(c) the physical environment in which he works
(d) the personal relationship he establishes with his colleagues and superiors
07. Which one of the following is correct? If, a person get more satisfaction from a particular
work, he will
(a) stick on the that work for every
(b) certainly achieve mastery over that work soon
(c) slowly develop a taste for that work (d) do that work in a better and harder manure

247
Academy

GENERAL ENGLISH

ACE

08. Which one of the following is correct? Both the scientists and the missionaries work
(a) because of the incentives given to them
(b) without hopping for any material gain
(c) without receiving any material gain
(d) because they want to be appreciated

09. Which one of the following is correct? Every person wants to have a work which
(a) helps him to get involved meaningfully
(b) helps him to earn substantially
(c) is pleasant and easy
(d) can be finished quickly
PASSAGE III
In our country there is very little popular writing on science. Those who write are writing for
other scientists. The newspapers these days devote a little more space then before for
scientific topics but they appear to be rather ill-digested knowledge not written in simple
readable language. So if we are to bridge this gap and disseminate scientific knowledge and
promote scientific temper, it has become necessary for some of the scientists to turn to
popularization. Today we have almost compulsion for doing this if we are to convey to the
people the meaning and relevance of the pursuit of science. It is time same scientists enters
the field of scientific journalism.
10. What does the passage suggest?
(a) The author is fully satisfied with the role of newspapers in promoting popular writing
on science.
(b) The author is totally dissatisfied with therole of newspapers in promoting popular
writing on science.
(c) The author appreciates the readiness of newspapers to allow more space than before to
writings on science.
(d) The author condemns the attitude displayed by newspapers in the matter of promoting
popular writing on science.
11. Which one of the following is correct? Popular writing on science in our country appears
to be
(a) not properly digested and easily comprehensible
(b) extremely restricted in outlook
(c) very lucid, intelligible and self explanatory
(d) based on obsolete knowledge of the topics chosen
12. Which one of the following is correct? The propose of promoting popular scientific
writing in newspapers is
(a) to attract more people to the study of science
(b) to spread scientific knowledge and encourage scientific temper among the common
people
(c) to give the people information about modern scientific inventions
(d) to help the newspapers to increase their circulation among the students of science!
13. The author wants some of the scientists to turn to popularization. What does that
imply?
(a) The scientists should go round and country and explain to the people the various

ACE Academy
248

COMPREHENSION

achievements of science J
(b) The scientists should make use of the television and radio to spread the message of
science among the common people
(c) The scientists should write in newspapers about the various aspects of science in
easily understandable language.
(d) The scientists should seek the help of social organizations to spread scientific
knowledge among the common people.
Keys:
01.d 02.b 03.d 04.a 05.c 06.b 07.d 08.b 09.a 10.c 11.a 12.b 13.c

2007
PASSAGE I
Fungi form an isolated group with in the plant kingdom and indeed are regarded by many as
forming a separate kingdom of their own. They differ from all other plants by their lack of the
green pigment chlorophyll, in the construction of the thread like units known as hyphae and
their method of reproduction.
Reproduction occurs by spores either of sexual or asexual origin. The spore germinates,
sending out a germ tube that elongates to produce a thread like usually septate, filament
that then branches out repeatedly. By continued ramifications, these threads or hyphae form a
cobweb or felt like sheet known as a mycelium. In most instances, fusion between two
hyphae, usually from different mycelia, must occur before a fruit body can be produced
Even then this process will only take place given the correct climatic factors and a sufficient
food supply. Fungi differ from the higher plant forms by the absence of the green pigment
chlorophyll, which enables plants to photosynthesize. By this process green plants are able to
obtain their carbohydrates; the chlorophyll in their leaves fixes atmospheric carbon dioxide in
the presence of sunlight and water to manufacture sugar for their nutrition. As fungi are
unable to do this they have to obtain their carbohydrates from decomposed animal or plant
tissues. Hence they are found in habitats rich in rotting vegetation such as woodland,
grassland, compost heaps, sawdust piles and on dung or manure heaps.
01. Consider the following statements:
Fungi grow on rotting vegetation or animal because
1. they cannot produce their own food
2. this provides them with the food they need
Which of the statements given above is/are correct?
(a) 1 only
(b) 2 only
(c) Both 1 and 2
02. What are fungi?
(a) Cells

(b) Plants

(c) Animals

(d) Neither 1 nor 2


(d) Dead matter

03. Consider the following statements:


1. Fungi are the green pigment called chlorophyll.
2. All plants other than fungi have chlorophyll.
Which of the statements given above is/are correct?
(a) 1 only
(b) 2 only
(c) Both 1 and 2
04. How do fungi reproduce?
(a) They reproduce from seeds
(b) Spores are the means for their reproduction
Which of the statements given above is/are correct?

(d) Neither 1 nor 2

249
Academy
(a) 1 only

GENERAL ENGLISH
(b) 2 only

(c) Both 1 and 2

ACE
(d) Neither 1 nor 2

05. How do fungi reproduce?


(a) They reproduce from seeds
(b) Spores are the means for their reproduction
(c) they reproduce through saprophytes
(d) they reproduce through decomposed animal or plant tissues

06. Consider the following statements:


Sugar cannot be produced by plants unless
1. Chlorophyll is present
2. Sun light, water and carbon dioxide is present
Which of the statements given above is/are correct?
(a) 1 only
(b) 2 only
(c) Both 1 and 2

(d) Neither 1 nor 2

Passage II
Today, the import duty on a complete machine is 35% for all practical purposes, whereas the
import duty on the raw materials and components ranges from 40% - 85%. The story does not
end here. After paying such high duties on components, once a machine is made; it is subject
to excise duty from 5% - 10%. At the time of sale, the machine tools are subject to further
taxation, i.e., Central sales tax or State sales taxes which range from 4% - 16%. This much of
the tax angle. Another factor which pushes to cost of manufacture of machine tools is the
very high rate of interest payable the banks ranging up to 22%, as against 4% - 7% prevailing
in other advanced countries.
The machine tool industry in India has an enviable record of very quick technology
absorption, assimilation and development. There are a number of success stories about how
machine tool builders were of help at the most critical times. It will be a pity, in fact a
tragedy, if we allow this industry to die and disappear from the scene.
It may be noted that India is a least 6000km away from any dependable source of supply of
machine tools. The Government of India has always given a great deal of importance to the
development of small scale and medium scale industries. This industry has also performed
pretty well. Today, they are in need of help from Indias machine tool industry to enable them
to produce quality components at reduced costs. Is it anybodys case that the needs of this
fragile sector will be met from a distance of 6000 km?
Then, what is to that the industry expects from the government? It wants a level playing field.
In fact, all of us must have a deep introspection and recognize the fact that the machine too
industry has a very special place in the country from the point of strategic and vital interests
of the nation.
07. Consider the following statements:
1. The machine tool industry has a very meager role to play in India
2. The performance of the small scale industry can be further improved with the help
from the Indian machine tool industry.
Which of the statements given above is/are correct?
(a) 1 only
(b) 2 only
(c) Both 1 and 2
(d) Neither 1 nor 2
08. Which of the following best explains the sentence It wants a level playing field?
The machine tool industry in India

ACE Academy
250

COMPREHENSION

(a) needs liberalized policy to import the desired components at a low cost
(b) needs land at subsidized rate
(c) needs electricity at subsidized rate
(d) wants to adopt novel marketing strategies for sales promotion
09. Which one of the following is the correct statement?
(a) The Government of India has taken due notice of the problems of the machine tool
industry
(b) The Government of India has not taken sufficient measures to help the machine
tool industry
(c) India should not waste its precious resources on the production of machine tools
(d) Banks in other countries are running in loss owing to a low interest rate
10. According to the passage, all the following factors are responsible for high cost of
machine tools in India, except
(a) sales tax
(b) excise duty
(c) higher duty on components (d) high profit of margin of the manufacturers
11. Why do small and medium scale industries look for help from Indias machine tool
industry?
1. To compete with the IT sector
2. To produce components at lower cost without sacrificing quality
Select the correct answer using the code given below:
(a) 1 only
(b) 2 only
(c) Both 1 and 2
(d) Neither 1 nor 2
12. If the banks rate of interest in India is made at par with that in the other advanced
countries,
how will the cost of manufacture of machine tools in India vary?
(a) It will go up by 22%
(b) It will go up by 4% to 7%
(c) It will remain the same
(d) It will decrease considerably
Keys:
01.c 02.b

03.b

04.b

05.d

06.c

07.b

08.a

09.b

10.d

11.b

12.d

2006
PASSAGE-I
The atmosphere is a mixture of several gases. These are about ten chemical elements which
remain permanently in gaseous form in the atmosphere under all natural conditions. Of these
permanent gases, oxygen makes up about 21 percent and nitrogen about 78 percent. Several
other gases, such as argon, carbon dioxide, hydrogen, neon, krypton, and xenon, comprise the
remaining 1 percent of the volume of dry air. The amount of water vapour, and its variations
in amount and distribution, are of extraordinary importance in weather changes. Atmospheric
gases hold in suspension great quantities of dust, pollen, smoke, and other impurities which
are always present in considerable, but variable amount.
The layer of the air next to the earth, which extends upward for about 16 km, is known as the
troposphere. On the whole, it makes up about 75 percent of all the weight of the atmosphere.
It is the warmest part or the atmosphere because most of the solar radiation is absorbed by the

251
Academy

GENERAL ENGLISH

ACE

earths surface, which warms the air immediately surrounding it. A steady decrease of
temperature with increasing elevation is a most striking characteristic. The upper layers are
colder because of their great distance from the earths surface and rapid radiation of heat into
space. The temperatures within the troposphere decrease about 3.5 per 1,000 feet increase in
altitude. Within the troposphere, winds and air currents distribute heat and moisture. Strong
winds, called jet streams, are located at the upper levels of the troposphere. These jet streams
are both complex and widespread in occurrence. They normally show a wave-shaped pattern
and move from west to east at velocities of 240 kmph, but velocities as high as 640 kmph
have been noted. The influences of changing locations and strengths of jet streams upon
weather conditions and patterns are no doubt considerable.
Above the troposphere to a height of about 80 Km is a zone called the stratosphere. The
stratosphere is separated from the troposphere by a zone of uniform temperatures called the
tropopause. Within the lower portions of the stratosphere is a layer of ozone gases which
filters out most of the ultraviolet rays from the sun. The ozone layer varies with air pressure.
If this zone were not there, the full blast of the suns ultraviolet light would burn our skins,
blind our eyes, and eventually result in out destruction. Within the stratosphere, the
temperature and atmosphere composition are relatively uniform.
The layer upward of about 80 km is the most fascinating but the least known of these three
strata. It is called ionosphere because it consists of electrically charged particles called ions,
thrown from the sun. Its effect upon weather conditions, if any, is as yet unknown.
01. It can be inferred from the passage that a jet plane will usually have its best average
speed on it run form:
(a) London to New York
(b) New Delhi to Tokyo
(c) New Delhi to Washington
(d) Melbourne to New Delhi
02. This passage does not provide information about:
(a) The effect of refrigerants on ozone depletion
(b) The amount of oxygen in the atmosphere
(c) The presence of considerable waste products in the atmosphere
(d) The presence of permanent gases in the atmosphere.
03. Which of the following quantities is/are answered on the basis of information contained
in the passage?
1. How do the troposphere and the stratosphere differ?
2. How does the ionosphere affect the weather?
Select the correct answer using the code given below:
(a) only 1
(b) only 2
(c) Both 1 and 2
(d) Neither 1 nor 2
04. It can be inferred from the passage that at the top of a location which is above 16,000 feet
above a town, the temperature is usually :
(a) Below freezing
(b) Warmer than that in the town
(c) About 56 colder than that on the ground
(d) Affected by the ionosphere
05. Life as we know it exists on the earth because the atmosphere:
(a) Is warmest at the bottom
(b) Carries the ultraviolet rays of the sun
(c) Contains a layer of ozone gases
(d) Contains a lot of water vapours
06. The troposphere is the warmest part of the atmosphere because it:

ACE Academy
252

COMPREHENSION

(a) Radiates heat into space


(b) Is warmed by the earths heat
(c) Has winds and air currents that distribute the heat. (d) Contains jet streams
Keys for 2006:
01.b 02.a 03.a

04.c

05.c

06.b

2005
PASSAGE I
Education, the most crucial investment in human development, is an instrument for
developing an economically prosperous society and for ensuring equity and social justice. A
holistic view was taken in 1986 when the National Policy on Education (NPE) visualized
education as a dynamic, cumulative, life long process, providing diversity of learning
opportunities to all segments of society. The Prime Ministers Special Action Plan (SAP) has
stressed the need for expansion and improvement of social infrastructure in the field of
education. The government has accorded high priority to education, the main facets of which
are (i) total eradication of illiteracy, (ii) gradual increase in the governmental and non
governmental spending on education up to 6 percent of GDP, (iii) implementation of the
Constitutional provision of making primary education free and compulsory up to V standard,
(iv) move towards equal access to and opportunity of education at all levels from primary
schools to universities.
01. Which one of the following statements is correct?
(a) Equity and social justice can be achieved only through industrial development in the
country.
(b) There is a Constitutional provision for free and compulsory primary education.
(c) The government spends up to about 6% of GDP on education.
(d) The prime Ministers SAP has stressed on the reservation for SC/ST students in the
universities.
02. Which one of the following statements is correct? The National Policy on Education has
taken cognizance.
(a) of poor quality of a large number of newly set up universities.
(b) poor teacher / student ratio in the primary schools.
(c) of the wide array of learning opportunities which education provides to the
society.
(d) of the need to provide vocational education.
03. Which one of the following statements is correct? The government, in its education
policy, has stressed
(a) on distance education.
(b) on e learning.
(c) on role of private sector in education.
(d) on access of education to all sections of society.
04. Which one of the following statements is correct? NPEs approach to education,
(a) prescribes a proactive role for the University Grants Commission.

253
Academy

GENERAL ENGLISH

ACE

(b) view it as a dynamic and continuous process.


(c) emphasizes on learning of English because of globalization.
(d) is based on the premise that education can be effectively imparted only up to 21 years
of age.

PASSAGE - II
Provision of quality infrastructure service at a reasonable cost, is a necessary condition for
achieving sustained economic growth. In fact, one of the major challenges being aced by the
Indian economy, as we enter the new millennium is to enhance infrastructure investment and
to improve the delivery system and quality of services. Government recognizes the critical
importance of the infrastructure sector and accords high priority to development of various
infrastructure services such as power, railways, roads etc. Investments in these sectors
involve high risk, low return, lumpiness of huge investment, high incremental / output ratio,
long payback periods, and superior technology. These prerequisites pose a constraint on the
Governments efficient delivery of quality infrastructure services. Government is moving
away from its traditional role as a provider of services to one of facilitator and regulator by
ensuring that infrastructure services are actually delivered in a desirable manner. While
liberalizing the rules and procedures, it has created an environment conducive for private
participation including foreign investment in infrastructure sector. The government however
continues to safeguard to interests of the consumers and needs of the poor by providing
appropriate regulatory framework.
05. Which one of the following statements is correct?
(a) Political considerations outweigh economic reasoning when it comes to development
of infrastructure sector.
(b) Developments in infrastructure sector are highly capital intensive.
(c) The government is not encouraging the foreign participation in development of
infrastructure sector.
(d) As per provisions of the Constitution of India, only public sector can participate in
development of infrastructure sector.
06. Which one of the following statements is correct?
(a) Government machinery has failed to deliver in the infrastructure sector.
(b) The government should only act as a provider of infrastructure services.
(c) Many corrupt practices have come to light during governments involvement in
infrastructure development.
(d) Various requirements of resource for infrastructure development are an
impediment for the governments role as a provider of infrastructure service.
07. Which one of the following statements in correct? Regulatory framework by the
government is required
(a) to raise capital for infrastructure development.
(b) for public distribution system.
(c) for a sustained economic growth.
(d) to ensure that economically weaker sections of the society do not suffer.
Keys for 2005:

ACE Academy
254
01.b

02.c

03.d

COMPREHENSION
04.b

05.b

06.d

07.d

2004

Passage I
These are dangerous times for conservationists in the Galapagos Islands, whose unique
ecosystem inspired Charles Darwins work on evolution. Last week 30 scientists were held
hostage by angry fishermen demanding the right to use semi-industrial techniques in the
protected waters around the archipelago. They blocked roads, burned tyres and threatened to
unleash goats on pristine islands. Ecuadors government quickly caved in. It promised to set
up a committee, on which the fishermen have many allies, to study their claims.
Unless reversed, this decision would have very serious implications for conservation, says
Sylvia Harcourt of the Charles Darwin Foundation, which runs a research station on the
islands. Since 1998, when Ecuador declared the Galapagos to be a marine reserve, decisions
about its management have been made by a committee on which conservation bodies, tourist
firms, government ministries and fishermen are all represented. The aim is to give the islands
18000 residents a say, while striking a balance between conservation and development.
01. Which one of the following statements is correct?
Fishermen in the Galapagos Islands were angry and they were demanding
(a) higher price
(b) better infrastructural facilities
(c) exemption from tax
(d) the permission to use advanced techniques
02. Which one of the following statements is correct?
Ecuadors government
(a) had steadfastly refused to listen to the demands of the fishermen
(b) was supporting the cause of the environmental scientists
(c) had promised to look into the grievances of the fishermen
(d) had arrested the leaders of the fishermen
03. Which one of the following statements is correct?
Galapagos Islands
(a) is known for water-skiing events
(b) is known for its exquisite beaches
(c) is known for its cashewnut production
(d) has been a unique nature reserve

Passage II
Coming to think of it, I missed the freedom fighters pension by a whisker. What a pity that I
only draw the Army officers pension.
In 1947, just before India got her freedom, as an eight-year-old lad. I made an impassioned
anti-British speech at a huge rally organized by Congressmen at our district headquarters. My
speech was short, it was by rote, and I made it standing on a table. But it was firebrand. My
father had detailed a teacher specifically to train me in the art of public speaking and to take

255
Academy

GENERAL ENGLISH

ACE

me to various anti-British meetings. Hats off to my father. Though a humble farmer he had
the vision that the future of his son lies in politics and not in farming.
Anyway, at the meeting, just as I made my fiery speech, there was a raid. People ran helterskelter and the police were all over the place. My good teacher whisked me away from the
scene of battle. Next day, news of the lathi charge on the speakers was in all the newspapers.
None of these speakers are alive today but their widows draw handsome pension from the
Consolidated Fund of India.
04. Which one of the following statements is correct?
(a) The author was saved from the lathi charge by a kind policeman
(b) Widows of the freedom fighters draw pension from the Contingency Fund of India
(c) There was a lathi charge on the speakers in the anti-British rally
(d) Authors father hired a teacher to make his son learn English
05. Which one of the following statement is correct?
(a) Author was an Army officer
(b) Author delivered an impromptu speech at the rally
(c) Authors father wanted his son to take to farming
(d) Author draws pension as a freedom fighter
06. Which one of the following statements is correct?
(a) Authors father was a teacher
(b) The anti-British rally was organized by Congressmen
(c) Authors father being illiterate had no vision about his sons future
(d) Authors father tried to get freedom fighters certificate for his son

Passage III
Sam Gedeon, a grocer a Sweden, was in his shop one night in April when a knife-yielding
teenager stormed in. Just a second, Gedeon told the boy, who demanded 100 Swedish
kronor about Rs.600. The money is upstairs.
Gedeon returned not with cash but with his Nokia 7650 mobile phone, which can take and
transmit digital photographs. He snapped a shot of the 15 year-old, who then started smashing
the register before taking off. I could have stopped him says Gedeon, but as long as he
didnt try to take the phone I didnt think it was worth fighting.
Gedeon forwarded the photograph to his computer. Thirty minutes later local police used a
printout to identify the teenager in a nearby caf. Camera phones capable of taking a photo
and transmitting it wirelessly may not have been designed with crime fighting in mind. But
its just one example of the diverse ways the phones have been used since their introduction
in Japan two years ago.
07. Which one of the following statements is correct?
(a) Gedeon used closed circuit TV to catch the teenager
(b) Gedeon used his computer also to help the police trace down the teenager
(c) The teenager made an abortive attempt to snatch Gedeons mobile phone
(d) Gedeon fired a shot at the teenager but the teenager escaped unhurt.
08. Which one of the following statements is correct?
(a) Gedeon had prior information about the teenager and had accordingly informed the
police
(b) Gedeons mobile phone used analog technology
(c) The incident in Gedeons shop proved the versatility of mobile phone
(d) Gedeons assistants helped him overpower the teenager

ACE Academy
256

COMPREHENSION

09. Which one of the following statements is correct?


(a) Gedeon felt that it was not worth fighting with a juvenile delinquent
(b) Mobile phones were introduced in Sweden
(c) Gedeon took a photograph of the teenager using his mobile phone
(d) The teenager took away the register from Gedeons shop
Keys for 2004:
01.d 02.c 03.d 04.c 05.a 06.b 07.b 08.c 09.c

2003
PASSAGE-I

Job enlargement stems from the thinking of industrial engineers. The idea is to
break up the monotony of a limited routine and work cycle by increasing a jobs scope. Work
functions from a horizontal slice of an organizational unit are combined, there by giving each
employee more operations to perform. For example, the work from two or more positions
may be combined to restore some sense of the wholeness of the job. Another basic strategy
was inspired by motivational theory. Job enrichment tries to deal with dissatisfied workers by
increasing the depth of their jobs. Work activities from a vertical slice of the organizational
unit are combined into one position to give employees more autonomy on the job. The idea is
to develop a stronger sense of accountability by allowing workers to set their own work pace,
correct their own errors, and decide the best way to perform various tasks. Workers may also
be shifted routinely from job to job within the same company so that they can develop a
variety of skills. Job rotation of this sort motivates workers by challenging them and enabling
them to learn new skills. Job rotation may be combined with job enlargement and job
enrichment.
01. Job rotation
(a) increases workers punctuality
(b) spurs the workers to learn new skills
(c) enables the management to reduce losses
(d) increases an organizations competitive edge
02. Job enrichment
(a) motivates the workers to decide the best way to perform different tasks
(b) enhances the quality-consciousness of the workers
(c) is best developed during brain-storming sessions
(d) helps the workers to get enhanced wages
03. Job enlargement is practiced mainly
(a) to increase volume of sales
(b) to combine work functions so as to increase jobs scope
(c) to reduce the manpower requirement
(d) to stimulate workers innovative thinking
04. Select the correct statement:
(a) Job enlargement increases the depth of the job for dissatisfied worker
(b) A basic premise of job enrichment is that it permits the workers to set their own work
pace

257
Academy

GENERAL ENGLISH

ACE

(c) Job enlargement and job enrichment are synonyms for the same approach
(d) Job enlargement and job rotation cannot be combined

PASSAGE-II
Our busy and absent-minded father would never worry about us children; our
mother did worry. Yet she allowed us to go off into the hills immediately after breakfast, and
did not complain when we came back long after supper-time. Though she had a bad head for
heights, she never restrained us from climbing in dangerous places; and we never got hurt.
Having a bad head for heights myself, I trained myself deliberately and painfully to overcome
it.
05. The children regularly went to the hills
(a) for the greater part of the day
(c) for the afternoon only
06. In this passage, restrained implies:
(a) Refused
(b) Suspected

(b) for a couple of hours


(d) after sunset still supper-time
(c) Forced

07. The phrase deliberately and painfully implies:


(a) Pleasure
(b) Fear
(c) Determination

(d) Held back


(d) Effort

08. The father did not worry about the children because
(a) he was involved in his business
(b) he knew their mother did
(c) he did not care about them
(d) men worry less than women do
09. Which of the following statements best expresses the main thought in this passage?
(a) The indifference of the parents led to the childrens growing up wild.
(b) The mother trusted her children and in spite of normal anxiety, gave them freedom to
explore the countryside and they came to no harm.
(c) His mothers fear of heights spurred on the author to conquer the same fear in
himself.
(d) The mother never complained when her children were climbing.
PASSAGE-III
As time passed, signs of age and weariness started showing in Galateas
appearance. This made Pygmalion very unhappy for he believed that beautiful creations must
remain permanent. Pygmalion foresaw her inevitable fate and when night came he took his
chisel and struck her bosom a blow and lo ! Galatea returned to the original marble. All that
night he chiseled. Next morning he saw something quite different compared to the original.
The lips had lost their great beauty; the eyes told of the grief of living; the whole body was
bent towards the earth. The whole night, in the dark, he had been sculpting the very face of
grief. And Pygmalion wept.
10. Galateas face started showing signs of
(a) satisfaction
(b) happiness
(c) age and weariness
(d) fun and frolic

ACE Academy
258

COMPREHENSION

11. Pygmalion believed like a true artist that


(a) perfect beauty was impossible
(b) beautiful creations must remain permanent
(c) ugliness was the real thing
(d) all art was meaningless
12. The artist reconverted Galatea to stone because
(a) he was jealous of her exceptional beauty
(b) he was tired of her
(c) he had fallen in love with another person
(d) she was growing old and haggard
13. Pygmalion re-chiselled Galateas statue
(a) to make it more beautiful
(b) to make it more real with a look of suffering and grief
(c) to make it look like a Goddess
(d) to give it a funny look
14. Pygmalion wept after recasting the statue because
(a) he had realized that human life was a drama of pain
(b) he was extremely upset at having destroyed his original creation
(c) he could not bear his self-inflicted loss
(d) he was happy and the tears indicated his joy.
PASSAGE-IV
March 12, 1992 Mauritius becomes a republic and is tipped as the first tiger of the
Indian Ocean and Africa. Population density is still high at 1500 persons per square mile but
the population growth rate has been contained at less than two per cent per year in the 1990s.
The structure of the economy has undergone substantial transformation. Diversification away
from sugar(but not at its expense) has been achieved. Textile and garment, and tourism are
the two other leading sectors today. The major challenge today is not finding jobs but finding
enough skilled labour to maintain the competitiveness of the economy. The unemployment
rate stands at less than three per cent. The overall balance of payments is positive. Income per
capita is Rs. 25000, placing Mauritius among the middle income group of countries.
Mauritius is now poised to move to a higher development phase.
15. The main industry of Mauritius still is
(a) textiles
(b) tourism

(c) sugar

(d) cement

16. One of the chief concerns of the country now is to


(a) find skilled workers
(b) find jobs for the unemployed
(c) diversified industrial growth
(d) change the economic structure
17. Which one of the following statements does NOT directly refer to the economic
development of the country?
(a) Population growth rate has been contained
(b) Industry has become competitive
(c) Income per capita is nearly Rs.25000
(d) It has become a Republic
18. The population growth rate per year in the 1990s is
(a) less than three per cent
(b) three per cent
(c) less than two per cent
(d) two per cent
19. That Maurities is tipped as the first tiger of the Indian Ocean and Africa suggests it
(a) can fight its own battles
(b) has developed a good image
(c) can help other countries solve their problems
(d) has transformed its environmental features

259
Academy

GENERAL ENGLISH

Keys for 2003:


01.b 02.a 03.b

04.b

05.a

06.d

07.c

13.a

16.a

17.d

18.c

19.a

14.b

15.c

08.a

09.c

ACE

10.c

11.b

12.d

2002
PASSAGE I
Gandhi has sound economic and cultural reasons for encouraging the revival of cottage
industries, but he does not counsel a fanatical repudiation of all modern progress. Machinery,
trains, automobiles and the telegraph have played important roles in his colossal life. Fifty
years of public service, in prison and out, wrestling with practical details and harsh realities
in the political world, have only increased his balance, open-mindedness, sanity, and
humorous appreciation of the quaint human spectacle.
01. Fanatical repudiation in this passage means
(a) unreasonable rejection
(b) mad pursuit
(c) senseless condemnation
(d) total rejection
02. Gandhi thought that machinery and various other modern scientific inventions
(a) were absolutely bad
(b) had played a significant role in his own life
(c) thwarted all his plans to encourage cottage industries
(d) were an anathema to him
03. Long spells of prison life and struggle for Indias freedom
(a) strengthened Gandhijis opposition to British Raj
(b) affected Gandhis way of thinking
(c) made Gandhi a balanced and cosmopolitan person
(d) made Gandhi an odd individual
04. Wrestling with practical details in this passage means
(a) fighting against the odds in daily life
(b) finding ways and means to defeat the enemy
(c) working out the details of the struggle for freedom
(d) grappling with the problems that confronted Indias struggle for freedom
PASSAGE-II
Human ways of life have steadily changed. About ten thousand years ago, man
lived entirely by hunting. A settled civilized life began only when agriculture was discovered.
From that time to this, civilization has always been changing. Ancient Egypt-Greece-the
Roman Empire-the Dark Ages and the Middle Ages-the Renaissance-the age of modern
science and of modern nations-one has succeeded the other, and history has never stood still.
Even if we try to do nothing, we can not prevent change.
05. The subject of the passage is
(a) the revolutionary process of the growth of mankind
(b) the biological evolution of mankind
(c) the revolutionary process of growth of civilization
(d) the dialectic process of civilization

ACE Academy
260

COMPREHENSION

06. The primitive man lived by


(a) domesticating animals
(c) hunting animals for food

(b) taming the wild animals of the forests


(d) gathering fruits from the forest

07. Human civilization began with the


(a) discovery of agriculture
(c) discovery of stars and planets

(b) discovery of sea-routes


(d) discovery of machines

08. The author emphasizes that civilization


(a) changes abruptly
(c) does not change without any reason

(b) changes constantly


(d) changes due to inherent contradictions

PASSAGE-III
A hundred years ago people wondered why anyone should be concerned about the
tiger. The tiger population was quite large, and the thick and widespread forest cover
provided a natural habitat for the animal. But over the years, poaching and increasing
encroachment on the tigers habitat have caused the tiger population to decline alarmingly in
India. To ensure its survival, the Indian Government has placed the tiger in the endangered
species list, and also declared it as Indias national animal.
09. A hundred years ago, people were
(a) very much concerned about the tiger
(b) not bothered about the tiger population
(c) fond of hunting tigers for pleasure
(d) afraid of tigers
10. The population of the tiger was large at that time because
(a) there was enough forest cover for them
(b) people left them undisturbed
(c) people did not hunt them so much
(d) the forest provided natural and easy food for them
11. The tiger population has come down today because of
(a) the declining health of tigers
(b) their lack of fertility
(c) insufficient availability of food for them
(d) killing of tigers and deforestation
12. The government has placed the tiger on the endangered species list
(a) to attract tourists
(b) to make the poaching of tigers difficult
(c) to ensure that tigers do not become extinct (d) to increase the population of tiger
PASSAGE IV
The author wants to make it quite clear that he took to the writing about inventors and
inventions mainly as a result of his exceedingly bad marks in Science, Mathematics,
Chemistry, and similar subjects at school. He could not understand any physical laws,
mathematical problems, and chemical formulae, and he regarded the people who understood
them, let alone discovered new ones, with boundless admiration. He wanted to find out what
goes on in the mind of an inventor or discoverer and how his inventions and discoveries
work. He tried to grasp these things with his unscientific brain, and, in making them clear to
himself in as simple a way as possible, found method of explaining them to other people like
himself, who had met with the same difficulties. This was how he began to write about
inventors and inventions.

261
Academy

GENERAL ENGLISH

ACE

13. The author took to writing about subjects concerning science because
(a) he was deeply interested in inventors and inventions
(b) new inventions and discoveries fascinated him
(c) he looked upon inventions and discoveries as an enigma
(d) he wanted to find out what motivates a scientist to invest or discover something
14. The author deemed the inventors to be
(a) crazy
(b) courageous

(c) highly talented

(d) interesting

15. The author decided to write this book to


(a) show his admiration for these inventors
(b) contribute to the growth of knowledge
(c) help lay man understand these matters in a single and clear manner
(d) inspire future inventors and discoverers
Keys for 2002:
01.a 02.b 03.b
13.c

14.c

04.d

05.c

06.c

07.a

08.b

09.b

10.a

11.d

12.c

15.c

2001
PASSAGE- I
Stress is a twentieth century illness. This is so because of the hectic pace of life and
the deadly rat-race around us. Different people react to stress in varied ways. Irritation, worry
depression are all signs of stress. It affects everyone from children to old people. Inability to
cope with it results in destructive tendencies. Of course, there is hope, because through some
people crack up, a lot of others dont. Hobbies, friends, music can help you reduce it.
01. According to the author, it is not the case that
(a) stress was known in earlier times
(b) one can cope with stress with some effort
(c) stress makes no distinction between young and old
(d) pleasurable activities help one get over stress
02. The word rat-race in the passage means
(a) extreme tension
(c) mad competition

(b) wild life


(d) terrible speed

03. The best way to cope with stress is to


(a) avoid getting irritated
(c) avoid all work and hectic activity

(b) fight destructive tendencies


(d) cultivate a few hobbies

PASSAGE -II
For generations Charlie Chaplin has been a source of amusement. His hero has to
struggle a lot with adversaries which are human as well as non human. For example, his
duel with the revolving door that always turns him out into the streets or the rocking chair

ACE Academy
262

COMPREHENSION

from which, once seated, he cant escape. This struggle has a deeper meaning. Not only
people but even objects defeat him because they are superior to him. His humanity cant
adapt itself to their mechanical nature. That is why we find him so close to us.
04. According to the author, Chaplins hero represents
(a) a silly man
(b) a funny person
(c) the common man
(d) the defeated man
05. The Chaplin hero cannot adjust himself to others because of their
(a) inimical behaviours
(b) indifference
(c) mechanical behaviours
(d) aggressiveness
06. According to the passage, Chaplins films represent the struggle between
(a) good and evil
(b) a human being and a hostile world
(c) a funny man and cruel machines
(d) a sane person and a mad world
07. The revolving door and the rocking chair in the passage are illustrations of
(a) Chaplins opponents in his imaginary world (b) natural forces which are hostile
(c) forces which curtail our freedom in society (d) Chaplins comic sense in his films

PASSAGE III
Lunar objects are well suited for observation when the sunlight falls upon them in such a
manner as to exhibit strongly contrasted lights and shadows. It is impossible to observe the
moon satisfactorily when it is full, for then no conspicuous shadows are cast. The mountain
peaks on the moon throw long, well defined shadows, characterised by a sharpness which
we do not find in the shadows of terrestrial objects. The difference between the two arises
from the absence of air on the moon. Our atmosphere diffuses a certain amount of light,
which mitigates the blackness of terrestrial shadows and tends to soften their outline. No such
influences are at work on the moon and the sharpness of the shadows is taken advantage of in
our attempts to measure the heights of lunar mountains.
08. Objects on the moon can be observed well
(a) when sunlight falls on them
(c) when light and shadows create a contrast

(b) at any time of the day


(d) only on a full moon night

09. A comparison between the shadows of lunar objects and those of the earth shows that
(a) they are more or less similar
(b) the shadows of lunar objects are sharper
(c) the shadows of terrestrial objects are sharper
(d) the shadows of lunar objects are not well defined
10. The difference between the two types of shadows is due to the fact that
(a) there is no air on the moon
(b) air creates sharper shadows of earthly objects
(c) the atmosphere of the moon intensifies the shadows of its objects
(d) more sunlight reaches the surface of the earth than that of the moon
11. The atmosphere surrounding the earth
(a) traps the sunlight thus intensifying its shadows
(b) scatters the sunlight thus intensifying its shadows
(c) dissipates the sunlight thus reducing the darkness of its shadows

263
Academy

GENERAL ENGLISH

ACE

(d) has no distinct function on reducing the darkness of its shadows

PASSAGE- IV
Gregory is about forty five and his hair is starting to go grey. Everybody knows
Gregory because he reads the news on television. He has done this for ten years and enjoys it
very much. He likes it when people stop him in the street or when they point at him and
whisper to their friends.
Yesterday his boss suggested that Gregory change his job. Gregory knows his boss wants a
younger man to take his place and doesnt care that happens to Gregory. The new job could
never be as good as his old one. He has no one to discuss the problem with at home and this
makes it worse.
12. People are familiar with Gregory because
(a) they can stop him in the street
(b) he is on TV everyday at news time
(c) he lives in the neighborhood
(d) he is happy when people talk about him
13. Gregory feels flattered when people
(a) cajole him for his news reading
(b) ignore all his drawbacks
(c) approach him to seek favours
(d) talk about him in appreciation
14. Gregory is likely to lose his job because
(a) he is not as popular as he used to be
(b) he has become very conceited
(c) television companies prefer younger people as newsreaders
(d) his boss does not like his work
15. Gregory is dissatisfied with his lot because his
(a) boss is unhappy with him
(b) boss is not concerned about his future
(c) boss has little regard for his loyalty
(d) hair has started to go grey
Keys for 2001:
01.a 02.c 03.d 04.c 05.a 06.b 07.c 08.c 09.b 10.a 11.c 12.b
13.d

14.c

15.c

2000
Passage I
For months they had barely kept themselves alive in a sort of shelter they built
with their own hands amidst the rubble. Then the German Elite Guard established
headquarters in Verona and for three dreadful years ruled the city with ruthless severity. The
boys grew to hate those harsh, unwanted masters and when the resistance movement began
secretly to form, they were among the first to join. It was not a matter of playing war. Their
extreme youth and insignificant size, added to an intimate knowledge of the neighboring
hills, made them immensely valuable.
01. The German Elite Guard came to the city to
(a) protect it from the attack by the enemy (c) arrest the criminals there
(b) provide food and shelter to the residents
(d) rule over the occupied land
02. What the boys joined was
(a) an association of gangsters
(b) the German Elite Guard
(c) a philanthropic organization engaged in relief work in the area

ACE Academy
264

COMPREHENSION

(d) a group working against the Germans


03. The boys were a great help to the resistance movement because
(a) they were good fighters
(b) they had an intimate knowledge of the hilly areas
(c) their movements were secret
(d) they hated their German masters
04. The passage conveys the impression that the
(a) city was ravaged by a war
(b) people were celebrating the liberation of the city
(c) people of the city had faced oppression
(d) young people hated their employer
Passage II
Ordinarily good memory is so common that we regard a man who does not possess
it as eccentric. I have heard of a father who, having offered to take the baby out in a pram,
was tempted by the sunny morning to pause on his journey and slip into a public house for a
glass of beer. Leaving the pram outside, he disappeared through the door of the bar. A little
later, his wife had to do some shopping which took her past the public house, where to her
horror she discovered her sleeping baby. Angry at her husbands behavior she decided to
teach him a lesson. She wheeled away the pram, picturing to herself his terror when he would
come out and find the baby gone.
How annoyed she was, however, when just before lunch her husband came in
smiling cheerfully and asking: Well , my dear, whats for lunch today ? having forgotten all
about the baby and the fact that he had taken it out with him. How many men below the rank
of philosopher would be capable of such absentmindedness as this? Most of us, I fear, are
born with prosaically efficient memories. If it were not so, the institution of the family could
not survive in any great modern city.
05. The writer is of the view that
(a) not many people have exceptional memory
(b) compared to women, men have very bad memory
(c) married men are often absentminded
(d) absentmindedness is a rarity while good memory is very common
06. The mother discovered her sleeping baby
(a) when she went out to search for it
(b) as she followed her husband knowing that he was absentminded
(c) accidentally when she went out shopping
(d) in the public house where her husband was having a drink
07. The mother wheeled away the pram without informing her husband because
(a) she was upset that he could be so irresponsible and callous
(b) she wanted to teach him a lesson
(c) she had never entered public houses
(d) she lacked presence of mind
08. The institution of the family survives because

265
Academy

GENERAL ENGLISH

ACE

(a) absentmindedness is a common feature


(b) good memory is a rarity
(c) most of us are able to remember our relationships
(d) mothers are more concerned than fathers

Passage III
Gautama subjected his body to exhausting fasts and every known form of
physical mortification, yet the truth he sought seemed distant as ever until one day, on waking
from a fainting fit brought on by his extreme asceticism, light came like a flash to his mind.
All the fating and self-hypnotism was leading him nowhere , was producing nothing but
increased enfeeblement of mind and body. To think clearly, a man must have proper food and
lead a healthy life.
Thrilled by his discovery, Gautama hastened to share it with his disciples who had
by now greatly increased in numbers. But they, when they heard their revered master
renounce the ascetic life and demand food, were so shocked and horrified by a suggestion
revolting to all their preconceived ideas, that to a man they deserted him. Left to himself,
Gautama wandered through the forests of Gaya, till he came to the banks of a river, where he
sat down to eat in the shade of a spreading bo-tree. And there, after enduring hour upon hour
of fiercest mental and spiritual anguish, after ranging through every emotion known to man.
Gautama found at last the peace and certainty he sought. Truth was revealed to him; he
became Buddha, the Enlightened One.
09. The passage suggests that
(a) only by torturing the flesh, one can discover the truth
(b) by living a prosperous and luxurious life alone, one can attain the power of clear
thinking
(c) self-inflicted suffering brought about by suppression of physical and mental
desires is not conductive to philosophical thoughts
(d) starving the body provides nourishment to the mind
10. Gautama wished to share the truth with his disciples, because
(a) he wanted to be known far and wide as a holy man
(b) his disciples had decided to desert him
(c) he wished his disciples to propagate his teachings
(d) he felt intensely excited about his discovery
11. Gautama became the Enlightened One
(a) on his attaining a vision of truth and inner peace after an intense emotional
experience
(b) on being left to himself after all his disciples had abandoned him
(c) after wandering in the woods of Gaya
(d) as a result of his going through physical and mental suffering
12. Gautamas disciples decided to desert him because they thought he had
(a) fallen in love with worldly pleasures
(b) broken his fast
(b) asserted that penance does not lead to enlightenment
(c) expressed his views against traditional thought and wisdom

ACE Academy
266

COMPREHENSION

Passage - IV
A local man, staying on the top floor of an old wooden house, was awakened at midnight by a
fire. Losing his way in a smoke-filled passage, he missed the stairway and went into another
room. He picked up a bundle to protect his face from the fire and immediately fell through
the floor below where he managed to escape through a clear doorway. The bundle proved to
be the bay of the Mayors wife. The hero was congratulated by all.
13. The man missed the stairway because
(a) he was extremely nervous
(b) he did not know where exactly it was
(c) he stumbled on a bundle
(d) the passage was full of smoke
14. The man picked up the bundle because
(a) it contained his savings
(b) the baby in it was screaming
(c) he thought it would prevent the smoke from choking him
(d) he thought it would keep the fire from burning his face
15. The man was called a hero because he
(a) showed great courage in fighting the fire
(b) rescued Mayors child
(c) saved a life
(d) managed to extinguish the fire
Keys for 2000:
01.d 02.d 03.b
13.d

14.d

04.c

05.d

06.c

07.b

08.c

09.c

10.d

11.a

12.c

15.c

1999
Passage I
When a man looks at a star, he sees the star not in its true position. The reason is that
the motion of the earth around the sun is carrying the observer through the space at a speed of
about 18.5 miles per second, so that the star-light he sees undergoes an apparent displacement
resulting from the combines effect of his velocity of the light. A similar phenomenon is
observed by a man driving a car at a moderate speed through a snow-storm at night. Even
though the snow may be falling vertically, it appears to be moving at an angle because of the
combined effect of its velocity and the cars
01. An observer is not able to see a star in its true position because
(a) the star is moving around the earth.
(b) he is standing still.
(c) the star-light seen by him undergoes a displacement.
(d) the star is too far away from the range of his vision.
02. The apparent displacement of star-light takes place because
(a) the star keeps moving with the earth at a velocity of 18.5 miles a second.
(b) the earth and the star-light both are moving at their own velocities.
(c) the light from the star takes a long time to reach the earth.
(d) the star is moving and its light takes little time to reach the earth.
03. To a man driving a car at a moderate speed in a snow-storm at night, the snow appears to
be falling
(a) horizontally
(b) vertically (c) both vertically and horizontally (d) at an angle

267
Academy

GENERAL ENGLISH

ACE

04. The experience of the man driving a car and the star-observer are comparable because of
(a) the rotation of the earth
(b) the fixity of the star
(c) their relative velocity displacement
(d) the raging of the storm

Passage-II
People who are serious about exploring their creative side have learned ways to heed and
preserve their new ideas. They have capturing skills. Salvador Dali, the great surrealist,
used to grab ideas for paintings from the fertile semi-sleep state called hypnagogic. Hed sit
in an armchair with a key in one hand and hold it over a plate placed on the floor. When he
drifted off to sleep, the sound of the key hitting the plate would awake him. Immediately,
hed sketch the bizarre images he was seeing.
05. In the passage, capturing skills refers to
(a) the ability to capture bizarre images
(b) the ability to chase and capture new ideas
(c) the creative abilities of a genius
(d) the ability to listen, understand and store new ideas.
06. Salvador Dali was
(a) a creative genius of ancient times
(c) an imaginative genius

(b) a well-known painter


(d) the founder of hypangogic

07. Hypnagogic refers to


(a) an active, half-asleep state
(b) a deep sleep full of dreams
(c) a disturbed state of half-sleep
(d) a manner of gathering ideas for paintings, during sleep
08. Salvador Dali used to wake him self up by
(a) relaxing on an armchair, instead of a bed
(c) the sound of the key hitting the plate

(b) holding a key in his hand


(d) dropping the key into the plate

Passage III
The heart is not just a mass of pumping muscle, but a hotbed of electrical activity, as
dependent on a stable supply of power as a computer chip is . When the ventricles, which
produce the hearts pumping action, do not get the right signals and beat inefficiently,
oxygen-rich blood does not reach the vital organs. In a heart with irregular heart beats - or
arrhythmia one short circuit can trigger another speeding the organ into tachycardia or
racing heart. In most severe cases, blacking out caused by ventricular fibrillation, during
which the heart quivers instead of beats, shock and sudden cardiac death follow.
09. From the passage, it is clear that the heart is
(a) a powerful muscle capable of producing electricity
(b) a supplier of power to the muscles
(c) an organ working on the principle of a computer
(d) a pumping muscle requiring continuous supply of power.

ACE Academy
268

COMPREHENSION

10. Oxygen-rich blood does not reach the vital organs when
(a) the ventricles beat irregularly
(b) the ventricles fail to give right signals
(c) arrhythmia results
(d) the heart receives inadequate power.
11. Ventricular fibrillation causes
(a) heart quivers instead of beats
(b) black outs, shock and death
(c) tachycardia or racing heart
(d) arrhythmia and short circuits.
12. It can be understood from the passage that what supports the heart most is
(a) hearts beats
(b) blood circulation
(c) ventricles
(d) muscles surrounding it
Passage - IV
I had better cells in other prisons, but in Dehradun I had one privilege which was very
precious to me. The gaol paper was a very small one, and we were kept in an old lock-up
outside the gaol walls, but within the gaol compound. This place was so small that there was
no room to walk about in it, and so we were allowed, morning the evening, to go out and
walk up and down in front of the gate, a distance of about a hundred yards. We remained in
the gaol compound, but this coming outside the walls gave us a view of the mountains and
the fields and a public road at some distance. This was not a special privilege for me; it was
common for all the A and B class prisoners kept at Dehradun.
13. The place where the narrator was kept
(a) was within the gaol walls and the gaol compound
(b) was outside the gaol walls, but within the gaol compound
(c) was so small that the could not walk long distances
(d) was within the gaol walls
14. The narrator was allowed to take a walk outside his cell every morning and evening
because
(a) he was advised to do so
(b) he was a political prisoner
(c) his cell was rather small
(d) he was a privileged prisoner
15. The privilege that the narrator had consisted of
(a) better facilities in the cell
(b) small and cosy room
(c) facilities for energizing walks and body-building physical exercises
(d) a fine view of the mountains and the life outside.
Keys for 1999:
01.c 02.b 03.d
13.b

14.c

04.c

05.b

06.c

07.d

08.c

09.d

10.b

11.b

12.b

15.d

1998

Passage I
There are a lot of things which are necessary before a person gets the chance to
think freely. For instance, he must have security; nobody can think about things, if he is
afraid of being robbed or murdered at any moment. Also he must have leisure to think in, and
he wont have that if he has to give all his attention to getting food to eat and clothes to wear,
if, that is to say, he spends all his time earning his living. And he must have other people to
talk to, so that you may say that security, leisure and society, which are all necessary to free
thinking , are necessary also to civilization.

269
Academy

GENERAL ENGLISH

ACE

01. A person should have enough leisure so that he


(a) can enjoy his food
(b) will have no tension in life
(c) can think in a relaxed manner
(d) will work for the welfare of the society
02. According to this passage, a person should have security in order to
(a) have an undisturbed sleep
(b) do his job in a relaxed manner
(c) lead a peace full life
(d) think in a free and frank manner
03. For the growth of civilization, it is necessary that people
(a) think and medicate in isolation
(b) are exposed to the history of civilized nations
(c) have congenial atmosphere for free discussion
(d) have absolute freedom

PASSAGE II
Is language, like food, a basic human need without which a child at a critical period
of life can be starved and damaged? Judging from the drastic experiment of Frederick II in
the thirteenth century in may be. Hoping to discover what language a child would speak if he
heard no mothertongue, he told the nurses to keep silent. All the infants died before the first
year. But clearly there was more than language deprivation here. What was missing was good
mothering. Without good mothering, in the first year of life especially, the capacity to survive
is seriously affected.
Today no such drastic deprivation exists as that ordered by Frederick. Nevertheless, some
children are still backward in speaking. Most often the reason for this is that the mother is
insensitive to the cues and signals of the infant, whose brain is programmed to mopo up
language rapidly. There are critical times, it seems, when children learn more readily. If these
sensitive periods are neglected, the ideal time for acquiring skills passes and they might never
be learned so easily again. A bird learns to sing and to fly rapidly at the right time, but the
process is slow and hard once the critical stage has passed.
04. According to the passage, a child can acquire language
(a) at any time of his life
(b) if adequate attention is paid to him during the sensitive period of learning
(c) only in the company of his mother
(d) only in the earliest period of his childhood.
05. The experiment of Frederick II revealed that language
(a) is very vital for a childs growth
(b) is a peripheral activity
(c) is more important for the child than mother care
(d) acts as a barrier in the growth of many other faculties
06. According to the passage, if mothers do not pay attention to the linguistic needs of their
infants,
(a) it can lead to permanent language loss
(b) several grammatical functions may never be acquired by them
(c) linguistic skills may be acquired with difficulty
(d) some children may develop serious mental problems
07. According to the passage, a human child is born with

ACE Academy
270

COMPREHENSION

(a) a special mechanism to learn a language fast


(b) no special mechanism to learn a language
(c) the same mechanism that birds and animals have
(d) a desire to learn a language

PASSAGE III
Deforestation and denudation in our country has already reached a serious stage. Unless the
forces and tendencies which are responsible for destroying the countrys environment are
checked in the near future and afforesatation of denuded areas taken up on a massive scale,
the harshness of the climatic conditions and soil erosion by wind and water will gradually
become impossible. The desert countries of the world and our own desert areas Rajasthan
desert is already on the march and is spreading into the adjoining states of Punjab, Haryana
and Uttar Prasad. Pockets of desert are appearing in other parts of the country including the
Himalayan region and the Deccan plateau. Where only a few decades back there used to be
lush green forests with perennial streams and springs, there is only brown earth, bare of
vegetation, without any water in the streams and springs except in the rainy season. The
ground water level is also falling noticeably.
08. According to the writer, deforestation and denudation will ultimately lead to
(a) the shortage of fuel for the common man
(b) lack of suitable land for cultivation
(c) the shortage or wood for furniture and building material
(d) decrease in wildlife
09. In order to control deforestation, the writer recommends that
(a) people who fell trees should be severely punished
(b) there should be large scale arrangement for irrigation in the forest
(c) forces and tendencies that are responsible for deforestation should be immediately
curtailed
(d) public opinion should be mobilized in favour or aforestation
10. The desert is Rajasthan in spreading
(a) all over Rajasthan
(b) into new areas in Rajasthan and Gujarat
(c) into Punjab, Haryana and Uttar Pradesh
(d) into the Deccan Plateau
11. Due to deforestation, in many areas in India perennial streams and springs have
(a) completely dried up
(b) scantly flow of water throughout the year
(c) irregular supply of water in summer (d) water only during the monsoon season

PASSAGE IV
Many songbirds learn to sing by listening to adult birds of the same species. If separate from
the adults, they develop unintelligible warbles rather than normal song patterns. But if taught
the song of another species, a bird often can pass the foreign language on the canarys song to
perfection. When it was later mated to a female bullfinch, its children and later its
grandchildren could sing like a canary.

271
Academy

GENERAL ENGLISH

12. Young songbirds learn to sing from


(a) their parents
(c) another bird of their species

ACE

(b) any singing bird


(d) any creature that can sing

13. Young birds can develop unintelligible songs when they


(a) listen to other song patterns
(b) listen to the birds of other species
(c) are separated from the adult birds of their of their own species
(d) are not in the company of other young songbirds
14. A songbird can pass the foreign language on to its offspring only when
(a) it listens to adults of the same species
(b) it learns a canary song
(c) it learns it from human beings
(d) it learns it from another species
15. The bullfinch could learn the canarys song because
(a) it was interested in it
(b) its was raised by male canary
(c) it mated a female canary
(d) it was raised by female canary
Keys for 1998:
01.c 02.d 03.c
13.d

14.d

04.b

05.d

06.c

07.c

08.b

09.c

10.c

11.d

12.a

15.c

1997

Passage I
The spectacle of a dying affects us painfully; we can see its struggles and, sympathetically
something of its pain. The unseen agony of a plant leaves us indifferent. To a being with eyes
a million times more sensitive than ours, the struggles of a dying plant would be visible and
therefore distressing. Boses instrument endows us with this more than microscopical
acuteness of vision. The poisoned flower manifestly writhes before us. The last moments are
so distressingly like those of a man that we are shocked by newly revealed spectacle of
them into a hitherto unfelt sympathy.
01. We are not moved by the agony of a plant because
(a) we are indifferent towards the plant.
(b) we do not see the agony of the plant
(c) we have no love for plants
(d) plants do not react like animals.
02. We can see the struggles of a dying plant
(a) with our naked eyes.
(b) with the help of a microscope
(c) with the help of Boses instrument
(d) if we have a sympathetic attitude towards it
03. In this passage the authors central point is that
(a) the plants suffer more than men
(b) we require an instrument to see the agony of plants
(c) our vision is too poor to see the agony of plants
(d) the plants are as sensitive to pain and pleasure as any other living being
04. The last moments of a dying plant are
(a) as agonizing as those of a dying animal
(b) more agonizing than those of a dying animal

ACE Academy
272

COMPREHENSION

(c) as ago nixing as those of a dying man


(d) less agonizing than those of a dying man

PASSAGE II
Many factors influence whether you notice a pain at all and, if you do, how much it hurts.
Since the pain message is received and interpreted by the brain, competing messages may
block it or other factors may enhance the intensity of the sensation. The pain threshold the
point at which a person reports can be raised as much as 40% by a hypnosis or by loud
noise or other distractions. Fully a third or pains can be relieved by a placebo, or sugar pill, if
the patient believes it to be an active pain killing drug. Recent studies suggest that placebos
work by triggering the release of the bodys own morphin endorphins. That a pain can be
dissipated by a placebo does not mean that it was feigned.
05. A pain may be felt lightly or intensely by a person according to
(a) his brain capacity
(b) how much it hurts
(c) the intensity of other messages to the brain
(d) the nature of the person and his attitude to pain
06. Loud noise is resorted to in reducing pain on the principle that
(a) it can shock a person out of consciousness
(b) it neturalises the cause of the pain
(c) the intensity of the pain sensation can be enchanced
(d) the brain can be distracted from niticing the pain message
07. According to the passage
(a) a placebo helps a doctor to detect whether a patient really has pain
(b) placebos help in relieving pain
(c) placebos immunize the body against pain
(d) doctors prescribe placebos in order to deceive their patients

PASSAGE III
Nearly all sports practiced nowadays are competitive. You play to win, and the game has little
meaning unless you do your utmost to win. On the village green, where you pick up sides and
no feeling of local patriotism is involved, it is possible to play simply for the fun and
exercise, but as soon as the question of prestige arises, as soon as you feel that you and some
larger unit will be disgraced if you lose, the most savage combative instincts are aroused.
Anyone who has played in a school football match knows this. At the international level sport
is frankly mimic warfare.
08. What is wrong with modern sports is that
(a) it has become very strenuous
(b) the sprit of competition has become excessive
(c) there is an undue stress on earning a lot of money
(d) it has become very technical
09. These days we play games
(a) just for entertainment
(c) to earn a lot of money

(b) just to pass time


(d) only to win

10. The savage combative instrincts of the of the players are aroused by

273
Academy

GENERAL ENGLISH

(a) the team spirit


(c) the hatred for other teams

ACE

(b) the element of prestige involved in winning a game


(d) the enthusiastic applause from the sudience

11. The game played at the international level


(a) makes the players more skilled
(b) creates international understanding
(c) creates the spirit of an international war
(d) is very rewarding even if the team loses the match

Passage IV
Difference between nations, so long as they do not lead to hostility, are by no means
to be deplored. Living for a time in a foreign country makes us aware of the merits in which
aour own county is deficient, and this is true whichever country our own may be. The same
thing holds of differences between different regions within one country, and of the differing
types produced by different professions. Uniformity of character and uniformity of culture
are to be regretted In the modern world, there is a real danger of too great similarity
between one region and another in cultural respects. One of the best ways of minimizing this
evil is on increase in the autonomy of different groups.
12. The author says that different between nations should be
(a) encouraged
(b) tolerated
(c) questioned

(d) suppressed

13. According to the passage, living abroad for a time


(a) makes us think of the merits of our own country
(b) leads us to forget the negative aspects of our motherland.
(c) makes us wonder if any other country is better than ours
(d) helps us to realize the drawbacks of our country
14. The regional difference within a county
(a) further the spirit of rivalry
(c) threaten national unity

(b) promote better mutual understanding


(d) control the fight for power

15. The author argues that uniformity of culture and character among individuals and groups
is
(a) harmful and undesirable
(b) unfortunate but unavoidable
(c) useful and unifying
(d) idleaistic but impractical
Keys for 1997:
01.b 02.c 03.d
13.d

14.b

04.c

05.c

06.c

07.c

08.b

09.d

10.b

11.c

12.b

15.a

1996
PASSAGE - I
The crouching position which the restricted spafce of his hiding place had forced
him to assume and the pain he still felt in the ankle made it seem like hours that his pursuers
searched in area where he lay concealed, although it was probably not more than a matter of
minutes. He dared not move; nor dared he give way to his overwhelming desire of sneeze for
fear or revealing his presence. When eventually the voices became fainter in the distance, he
allowed himself to shift his position and finally, when he could hear no more, to crawl, halfparalysed, from between the two rocks in whose shelter he had managed to escape capture.,

ACE Academy
274

COMPREHENSION

Cautiously he peeped out to see if it was safe for him to continue his escape, and ,
discovering no sign of life about him, started to creep painfully in the direction he had come,
hoping in this way to deceive his pursuers. He had not gone above a hundred yards when a
movement in the undergrowth nearby made him stand stockstill. Could it be that after all
some of his enemies were still searching the neighbourhood? A few seconds passed and then,
from behind a stump of a tree, there appeared a small black and white dog. Man and dog eyed
each other suspiciously for a moment, until, to the mans immense relief, it rushed up to him
wagging its tail.
01. The writer crouched in his wagging its tail.
(a) he did not want to take any risk of being caught
(b) he had pain in his ankle
(c) he was mortally scared
(d) there was not much space
02. When eventually the voices became fainter in the distance means
(a) it was relatively quiet now
(b) people talking in the distance fainted
(c) there was no sign of any activity nearby
(d) though there was some noise nearby, things wete quiet in the distance
03. The writer decided to come out of his hiding place when
(a) he was comopletely paralysed
(b) he could hear no more the voices of his pursuers
(c) he waw no longer able to control himself
(d) he was not able to see the pursures.

Passage II
Male lions are rather reticent about expending their energy in hunting-more than
three-quarters of kills are made by lionesses. Setting off a dusk on at hunt, the lionesses are in
front, tensely scanning ahead, the cubs lag playfully behind and the males bring up the rear,
walking slowly, their massive heads nodding with each step as if they were bored with thw
whole matter. But slothfulness may have survival value. With lionesses busy hunting, the
males function as guards for the cubs, protecting them particularly form hyenas.
04. According to the passage male lions generally do not go for hunting because
(a) they do not like it
(b) they want lioness to get training
(c) they wish to save their vigour for other things
(d) they are very lazy
05. When lionesses go in search for their prey, they are very
(a) serious
(b) cautious
(c) playful
(d) sluggish
06. Male lions protect their cubs
(a) from the members of their own species
(b) from hyenas only
(c) from hyenas as much as from other enemies
(d) more from hyenas than from other animals

PASSAGE - III

275
Academy

GENERAL ENGLISH

ACE

Harold, a professional man who had worked in an office for many years, had a
fearful dream. In it he found himself in a land where small slug-like animals with slimy
tentacles lived on peoples bodies. The people tolerated the loathsome creatures because after
many years they grew into elephants which then became the nations system of transport,
carrying everyone wherever he wanted to go. Harold suddenly realized that he himself was
covered with these things, and he woke up screaming. In a vivid sequence of pictures this
dream dramatized for Harold what he had never been able to put into words; he was himself
as letter society feed on his body in his early years so that it would carry him when he retired.
He later threw off the security bug and took up freelance work.
07. The statement that he later threwoff the secutiry bug means that
(a) Harold succeeded in overcoming the need for security
(b) Harold stopped giving much importance to dreams
(c) Harold started tolerating social victimization
(d) Harold killed all the bugs troubling him
08. Which one of the following phrases best helps to bring out the precise meaning of
loathsome creatures?
(a) Security bug and slimy tentacles
(b) Fearful dream and slug-like animals
(c) Slimy tentacles and slug-like animals (d) Slug-like animals and security bug
09. In his dream, Harold found the loathsome creatures
(a) in his village
(b) in his own house
(c) in a different land

(d) in his office

10. Harolds dream was fearful because


(a) it brought him face to face with reality
(b) it was full of vivid pictures of snakes
(c) he saw hugh elephants in it
(d) in it he saw slimy creatures feeding on peoples bodies
PASSAGE - IV
Laws of Nature, are not commands but statements of facts. The use of the word law in this
context is rather unfortunate. It would be better to speak of uniformities of Nature. This
would do away with the elementary fallacy that a law implies a law-giver. If a piece of matter
does not obey a law of Nature it is not punished. On the contrary, we say that the law has
been incorrectly stated.
11. Laws of Nature differ from man-made laws because
(a) the fomer state facts of Nature
(b) they must be obeyed
(c) they are natural
(d) unlike human laws, they are systematic
12. The author is happy with the word law because
(a) it connotes rigidity and harshness
(b) it implies an agency which has made them
(c) it does not convey the sense of Natures uniformity (d) it gives rise to false beliefs
13. If a piece of matter violates Natures law, it is not punished because
(a) it is not binding to obey it
(b) there is no superior being to enforce the law of nature
(c) it cannot be punished
(d) it simply means that the facts have not been correctly stated the law

ACE Academy
276

COMPREHENSION

14. The laws of Nature based on observation are


(a) conclusive about the nature of the universe
(b) true and unfalsifiable
(c) figments of the observers imagination
(d) subject to change in the light of new facts
Keys for 1996:
01.d 02.c 03.b
13.d

04.c

05.b

06.d

07.a

08.a

09.c

10.d

11.d

12.c

14.b

1995
PASSAGE - I
The joy of discovery is a very real incentive to research, despite the rareness of its
realization. It is an error to suppose that the scientist in unemotional, or could succeed if he
were. The error has arisen through a misconception. The absolute necessity that a scientists
findings shall not be changed from objective truth in response to emotional urges of any kind
does not result in his becoming a particularly unemotional person: whether a discover or
anyone else is pleased with a discovery has no effect on its validity. I have been working like
a madman at Drosen, wrote Darwin to Sir J. D. Hooker in reference to his study of
insectivorous plants and a few days later, to the geologist, Lyell at the present moment I care
more about Drosera than the origin of all the species in the world ---- I am frightened and
astounded at my results. Kropotkin once write, There are not many joys in human life equal
to the joy of the sudden birth of a generalization ---- He who has once in this life experienced
this joy of scientific creation will never forget it.
01. The author of the passage counters the misconception that a scientist is unemotional by
pointing out that
(a) the objectivity of a scientist cannot make emotional compromises
(b) even such a great scientist as Darwin cared so much for Drosera
(c) he has the capacity to react to emotional appeals made by others
(d) he considers scientific discovery equal to the joy of creation
02. In this passage the author tells us that scientific research leads one to
(a) rare discoveries
(b) sure success
(c) success, sometimes
(d) finding the objecting truth
03. Darwins letter to Lyell reveals to us that he
(a) attached greater importance to his discovery of insectivorous pants than to his
discovery of the origin of the species
(b) used to get deeply involved in whatever research, big or small, he had on hand at a
given point of time
(c) was rather uncertain about his success in his research on insectivorous plants
(d) was worried that insectivorous plants would destroy all species of insects
04. The findings of scientific researchers
(a) are universally welcome
(b) result in robbing the researcher of all emotions
(c) are objective in nature
(d) have universal validity
PASSAGE - II

277
Academy

GENERAL ENGLISH

ACE

One often comes across the remark, Its not what you know that gets you ahead; its
who you know. We do not subscribe to this cynical remark, but neither do we wish to
minimize the importance of having influential people back you up in your application. Often
the recommendation of a person whose word is respected is a deciding factor in getting a job.
This is perhaps especially true when an employer is considering a number of applications
from graduates who do not have professional experience. The academic records alone might
not provide a clear basis for a choice.

05. The recommendation letter of an influential person


(a) mattress much because it has come from someone important
(b) proves the candidates ability better than his or her academic record
(c) significantly helps where academic records are not decisive
(d) provides a clear basis, for selection to a job
06. A recommendation letter is important because, it
(a) is given by an influential person
(b) helps an employer to oblige an influential person be appointing the person
recommended by him or her
(c) helps an employer to choose from many who are not otherwise distinctive
(d) shows ones high connections
07. The writer does not affirm; he makes a tentative statement by using expressions like
(a) cynical remark, professional experience.
(b) We do not subscribe, neither do we wish---
(c) What you know-- who you know---
(d) often, perhaps, espically, alone
PASSAGE - III
I had hardly settled to work on the next issue of the morninger when the boss stormed
into the room, raging. Having joined his tabloid only a couple of months ago. I had come to
think of him as an urban, upright and meticulous mean. The outburst, caused by a minor slip
in the report filed by me yesterday, unnerved me completely. Was this the man who had
patted me on the shoulder only the other day for covering the Prime Ministers visit to the
state?
I asked myself in vain. I wrote out my resignation and walked out of the office in a
stupor.
08. The boss stormed into the room, raging, because
(a) he was sure that something terrible had happened
(b) the author had made some mistake
(c) he came to know that the author had come late
(d) the author had not carried out his orders
09. From reading the passage one can conclude that the author worked in
(a) a government office collecting and disseminating information about the visits of
ministers and other VIPs
(b) one of the departments of a privately-owned daily newspaper
(c) a private company bringing out a weekly news magazine
(d) a publicity and public relations office run by a ruling political party
10. The author left the office in a stupor because
(a) the sudden display of temper by the boss had unsettled him completely

ACE Academy
278

COMPREHENSION

(b) he had to resign his position because of a minor slip on his part.
(c) he felt that he had failed utterly in the task assigned to him
(d) the boss behaviour towards him had often been harsh
11. The boss of the establishment where the author worked was a/an
(a) nice man who insisted on perfection in work
(b) quarrelsome man quick to find fault with others work
(c) decent man
(d) unpleasant man of capricious moods
PASSAGE - IV
Then a terrible thing happened. While we were waiting for the coffee, the head
waiter, with an ingratiating smile on his false face, came up to us bearing a large basket full
of huge peaches. They had the blush of an innocent girl; they had the rich tone of an Italian
landscape. But surely, peaches were not in season then? Lord knew what they cost. I knew
too-a little later-for my guest, going on with her conversation, absent-mindedly took one.
You see youve filled your stomach with a lot of meat my one miserable little
chop and you cant eat more. But Ive just had a snack and I shall enjoy the peach.
The bill came, and when I paid it I found that I had only enough for a quite
inadequate tip. Her eyes rested for an instant on the three francs I left for the waiter, and I
know that she thought me mean. But when I walked out of the restaurant. I had the whole
month before me and not a penny in my pocket
12. Which one of the following words/groups of words suggests that the peaches were
attractive?
(a) Huge
(b) In season
(c) Blush of an innocent girl
(d) Ingratiating
13. Which one of the following statements about the passage is true?
(a) The incident happened at the authors home
(b) There were a number of guests
(c) The incident happened at a hotel
(d) The writer enjoyed the party
14. The author knew that the peaches would be expensive because
(a) they appeared huge
(b) they looked as beautiful as the Italian landscape
(c) the lady liked them
(d) they were difficult to come by at that time of the year
15. The lady might have thought him mean because
(a) he left three Francs as tip
(b) he did not eat any peach
(c) he had filled his stomach with a lot of meat
(d) he walked out rather abruptly
Keys for 1995:
01.b 02.a 03.c
13.c

14.d

04.c

05.c

06.c

07.a

08.b

09.b

10.c

11.a

12.c

15.a

1994
PASSAGE I
Some prophets of doom assert that we shall soon exhaust the Earths resources or
pollute ourselves to death. Optimists assert that Earths systems are robust and that improved
technologies will ease all the pressures on the plant. Others see the main concerns as political,
with environmental issues carrying the seeds of inequity and war.

279
Academy

GENERAL ENGLISH

ACE

Scientific understanding is too limited to say where the truth lies. If the care of the
planet is man agement task our species in the position of a child who has to fly a jet plane
without knowing what all the switches and levers do.
01. The blame for the environmental crisis facing the Earth lies with
(a) mans greed and selfishness
(b) inadequate management of Earths resources
(c) rapid technological growth
(d) pressures of over population
02. Which one of the following statements in true according to the passage?
(a) Sometimes political motives influence peoples concern for the planet
(b) Wars will cause greater pollution on Earth
(c) People will feel exhausted owing to the green house effect
(d) Men should feel responsible towards society
03. What does the comparison in the last sentence of the passage convey?
(a) We, on Earth are inexperienced in managing Earths resources
(b) Most of the people do not know how to fly an aero plane
(c) Our efforts to save the planet are childish
(d) We are as ignorant as infants
04. According to the passage nobody knows the truth because
(a) people do not know of what the future holds for them
(b) scientists often are used by politicians
(c) there are very few scientists capable of research in this area
(d) objective analyses are not thorough or exhaustive

PASSAGE II
I fell the wall of the tunnel shiver. The master alarm squealed through my
earphones. Almost simultaneously, jack yelled down to me that there was a warning light on
Fleeting but spectacular sights snapped into and out of view, the snow shower of debris,
the moon, looming close and big, the dazzling sunshine for once unfiltered by layers of air.
The last tweleve hours before recently were particularly bone chilling. During this period, I
had to go up into the command module. Even after the fiery re- entry and splashing down up
810 water in the South Pacific, we could still see our frosty breath inside the command
module.
05. The word, Command module used twice in the given passage indicated perhaps that it
deals with
(a) and alarming journey
(b) a commanding situation
(c) a journey into outer space
(d) a frightful battle
06. Which one of the following reasons would one consider as more possible for the warning
lights to be on?
(a) There was a shower of debris
(b) Jack was yelling
(c) A chatastrophe was imminent
(d) The moon was looming close and big
07. The statement that the dazzling sunshine was for once unfiltered by layers of air means
(a) that the Sun was very hot
(b) that there was no strong wind
(c) that the air was unpolluted
(d) none of the above

ACE Academy
280

COMPREHENSION

PASSAGE - III
The low unit of gas is a real temptation to any one choosing between gas and
electronic processes. But gas fired processes are often less efficient, require more floor
space, take longer and produce more variable product quality. These drawbacks negate the
savings many businesses believe they make.
By contrast, electricity harnesses a unique range of technologies unavailable with gas.
And many electric processes are well 90% efficient, so far less energy is wasted. With
benefits in terms of product quality and overall cleanliness, it can so often be the better and
cheaper choice. Isnt that tempting?
08. The passage can be described as
(a) an advertisement for electricity and its efficiency
(b) an extract from a science journal
(c) an appeal not to use gas
(d) an account of the growth of technology
09. Electricity harnesses a unique range of technologies What does the writer mean?
Electricity
(a) has developed new technologies
(b) ensures power for several technologies
(c) makes use of several technologies
(d) depends on new kinds or technology
10. What does the write mean by variable quality?
(a) Products from gas fired processes are inefficient
(b) The quality of the product cannot the assessed
(c) The quality of the products is not same
(d) The kind of products vary form time to time

PASSAGE IV
Religion is the greatest instrument for so raising us. It is amazing that a person not
intellectually bright, perhaps not even educated, is capable of grasping, and living by
something so advanced as the principles of Christianity. Yet that is a common phenomenon. It
is not, however, in my province to talk about religion, but rather to stress the power which
great literatue and the great personalities whom we meet in it and in history have to open and
enlarge our minds, and to show us what is first rate in human personality and human
character by showing us goodness and greatness.
11. The phrase so raising us means
(a) making us realize that we all are children God
(b) improving our mental abilities
(c) making us feel that we are more important than we really are
(d) giving us a sense of spiritual superiority
12. According to the author, we come across examples of greatness and nobility in
(a) books on Christianity
(b) historical records
(c) literary and historical works
(d) great works of literature
13. In the passage the authors ultimate intention is to talk about
(a) education
(b) character
(c) religion
14. What surprises that author is that
(a) Christianity is practiced by a large number of people

(d) history

281
Academy

GENERAL ENGLISH

ACE

(b) even uneducated people are attracted towards Christianity


(c) despite being difficult and complex, the principles of Christianity are practiced by so
many people
(d) even very intelligent people cannot understand the principles of Christianity
15. The author hesitates to talk about religion because
(a) he does not fully understand its importance
(b) he does not believe in any religion
(c) nobody around him likes to talk about it
(d) he does not feel himself competent to talk about it
Keys for 1994:
01.a 02.a 03.a 04.d 05.c 06.c 07.d 08.b
13.a

14.c

09.c

10.c

11.a

12.c

15.b

1993
PASSAGE - I
The last half of my life has been lived in one those painful epochs of human history
during which the world is getting worse, and past victories which had seemed to be definitive
have turned out to be only temporary. When I was young, Victorian optimism was taken for
granted. It was thought the world by an orderly process, and it was hoped that cruelty
tyranny, and injustice would continually diminish. Hardly anyone was haunted by the fear of
great wars. Hardly anyone thought of the nineteenth century as a brief interlude between past
and future barbarism.
01. A brief interlude between past and future barbarism can be interpreted as
(a) a short period of time between past and future acts of savagery
(b) a short space of time between two great events
(c) an interval between cruel wars
(d) a dramatic performance during wars
02. During the Victorian age people believed that
(a) strife would increase
(b) there would be unlimited freedom
(c) wars would be fought on a bigger scale
(d) peace could prevail and happiness would engulf the whole world
03. The word definitive as used in the passage means
(a) defined
(b) final
(c) temporary
04. The victories of the past
(a) brought permanent peace and prosperity
(b) ended cruelty, tyranny and injustice
(c) proved to be temporary events
(d) peace could prevail and happiness would engulf the whole world
05. The author feels sad about the later part of his life because
(a) he was nostalgic about his child
(b) the world had not become prosperous
(c) the author had not on any further victories
(d) the world was painfully disturbed during that period of time
PASSAGE - II

(d) incomplete

ACE Academy
282

COMPREHENSION

Until very recently, it was universally believed that men are congenitally more
intelligent than women; even so enlightened a man an Spinoza decided against votes for
women on this ground. Among white men, it is held that white men are by nature superior to
men of other colors, and especially to black men; in Japan, on the contrary, it is thought that
yellow is the best colour. In Haiti, when they make statues of Christ and Satan, they make
Christ black and Satan white. Aristotle and Plato considered Greeks so innately superior to
barbarians that slavery was justified so long as the master was Greek and the slave barbarian.

06. The author believes that


(a) Some colours are superior to other colours.
(b) Some colours are inferior to other colours
(c) Colours have no relevance to superiority
(d) Colours vary from country to country
07. Aristotle and Plato supported slavery because they thought
(a) The Greek to be superior to barbarians
(b) The barbarians to be superior to Greeks
(c) Slaves to be inferior
(d) That the barbarian belonged to Greece
08. In Haiti, Christs against votes for women because according to him
(a) Christ was evil
(b) Satan was good
(c) Black was good
(d) All white me were evil
09. Spinoza decided against votes for women because according to him
(a) They were less educated than men
(b) They were generally unintelligent
(c) They were naturally less gifted with intelligence
(d) They did not deserve to have votes
10. Congenitally means
(a) Certainly
(b) innately
(c) fantastically

(d) falsely

PASSAGE - III
The roots of the tree were rapidly losing their hold. The crow must have known that
something was wrong, because it kept flying up and circling the tree, reluctant to settle in it
and reluctant to fly away. As long ass the nest was there, the crow would remain flapping
about and cawing in alarm. Sitas wet cotton dress clung to her thin body. The rain ran down
from her long black hair. It poured form every leaf of the tree. The crow, too, was drenched
and groggy.
11. When the roots of the tree were rapidly losing their hold the crow was
(a) Unwilling to fly away
(b) Unable to fly away
(c) Indifferent to fly away
(d) Too groogy to fly away
12. The crow was reluctant to settle in the tree because
(a) It had instinctive sense of the trees impending fall
(b) Its nest had been wiped out
(c) It was drenched and groggy
(d) she felt friendship with the crow
13. Sita couldnt move away because

283
Academy

GENERAL ENGLISH

(a) The tree was still standing


(c) She felt friendship with the crow

ACE

(b) There was no other shelter


(d) She was nervous

14. The crow was reluctant to fly away because


(a) Its nest was in the tree
(b) It felt friendship with Sita
(c) If felt loyal to Sita
(d) It could not fly far away
15. The roots of the tree were rapidly losing their hold because
(a) There was heavy rainfall
(b) It was very old
(c) There was fierce storm
(d) It was on the edge
Keys for 1993:
01.a 02.d 03.b 04.c 05.b 06.c 07.a 08.d 09.c 10.b
13.b

14.a

11.c

12.a

15.a

1992
PASSAGE I
India is not a backward country. It is the version of the outside world which is
outdated. India is growing everyday. It has a population of more that 100 million in the
middle class, out of which a segment of 70 million is earning monthly wages more that the
average weekly pay of an Australian worker, which makes India an economic power as great
as France.
01. What is mean by segment in the above paragraph?
(a) the total population
(b) a large part of the population
(c) some part of the population
(d) half of the population
02. What is the strength of the Indian middle class?
(a) More than 100 million
(b) equal to 100 million
(c) less than 100 million
(d) 70 million
03. What is the underlying them of the paragraph?
(a) India is a great military power
(b) India is a backward country
(c) India is fast developing into a big power (d) India has poor economy
04. What is the meaning of the line India is as great as France?
(a) India is small as compared to France (b) India is greater than France
(c) India is on the same level as France
(d) India cannot be compared with France.
PASSAGE - II
By the first century AD, many nomadic tribes had entered India through the
mountain passes in the North West. In course of time, they were all Indianised. Kanishaka,
the chief of one of these tribes, built up an empire with Peshawar as the capital. Like Ashoka,
Kanishka came under the influence of Buddhism and was converted by a celebrated Buddhist
scholar Asvaghosha. Kanishaka appears to have tried to emulate Ashoka. He strenuously
worked for the spread of the Buddhist faith. He convened a Buddhist council to discuss grave
religious problems. At the time, a big schism had arisen and the Buddhists were divided into
two seets the Hinyana and Mahayana. Of the two, the Mahayana became more popular. The
Hinayanists believed in the traditional intellectual doctrine whereas the Mahayanists adopted
new interpretations. The Mahayanists started worshipping Buddha as the living savior and
their ritualism had a strong appeal to the ordinary people. Nagarjuna, a great scholar in the
court of Kanishka formulated and propagated the Mahayana ideas. The differences between

ACE Academy
284

COMPREHENSION

the two sots were largely reconciled in this council. Charaka, the most celebrated medical
scientist of ancient India, is believed to have been Kanishkas court physician. The Kushan
empire built by Kanishka was dissolved by the third century AD.
05. Which of the following statements is NOT true about Kanishka?
(a) He was the chief of a nomadic tribe.
(b) He was converted to Buddhism by Nagarjuna.
(c) Charaka was Kanishkas court physician
(d) He tried to emulate Ashoka.
06. The Mahayana sect became more popular because
(a) Kanishka was converted to this sect of Buddhism
(b) Kanishka strenuously worked to propagate it
(c) It believed in the traditional intellectual doctrine
(d) None of these.
07. Which of the following statements is NOT true in the context of the passage?
(a) Ashvaaghosha was a scholar
(b) Kanishka was a patron of Buddhism
(c) Hinayanists countered the traditional ideal of Buddhism
(d) Charaka was a doctor.
PASSAGE - III
It is no doubt true that we cannot go through life without sorrow. There can be no
sunshine without shade. We must not complain that roses have throns, but rather be grateful
that thorns bear flowers. Our existence here is so complex that we must expect much sorrow
and suffering. Many people distress and torment themselves about the mystery of existence.
But although a good man may at times be angry with the world, it is certain that no man was
never discontented who did duty in then to look at the bright side of things. Life has been a
comedy to those who think, and a tragedy to those who feel. It is indeed a tragedy at times,
and comedy very often, but as a rule, it is what we choose to make it.
08. We should expect much sorrow and suffering in life, because
(a) every rose has a thorn
(b) we think too much
(c) our life is so comlex
(d) Our own actions are responsible for them.
09. What do people distress and torment about?
(a) Life consists of both joys and sorrows (b) The world is like a mirror
(c) Life is a tragedy at times
(d) Secrets of life unknown to map.
10. What sort of man is never dissatisfied?
(a) Who does his duty in the world
(b) Who never gets angry with the world
(c) Who only looks at the bright side of things
(d) Who has no complaint about roses having thorns.
Keys for 1992:
01.c 02.a 03.c

04.c

05.b

06.d

07.d

08.c

09.d

10.a

1991

PASSAGE - I
Small car means less power, means less fuel consumption, less fuel consumption
means moir saving of the fuel which could be used for some other puroses for public

285
Academy

GENERAL ENGLISH

ACE

utilization. Thus, less for more should be the criteria among the Indians. Simple wastage
would mean no saving and the time has come review our expenses etc.
01. According to author less for more means
(a) getting bigger profit for bigger investment
(c) achieving glories without effort
02. Our expenses need a review because
(a) economy is the call of the day
(c) we have to save for tomorrow

(b) higher output for lesser input


(d) higher output for higher input
(b) wold war can erupt at any time
(d) none of the above

PASSAGE - II
Gift can necessarily be having costlier items such as gold etc, but even a post card
would do to complete its meaning. There is a golden rule for gift- What we dont want
should not be given as a gift to anybody. If we follow the same we would be having clearer
choice against three choices.
03. The golden rule of gift is
(a) difficult to follow
(c) having more worth

(b) easier to follow


(d) that it is a mere formality

04. The postcard in passage means


(a) a message of good wishes
(c) a photograph printed on a post card

(b) a cheapest gift


(d) an unusecd stationary which can be
further sued.

05. Costiler gifts


(a) not liked by receives due to threat from income tax authorities
(b) represent the financial position of the persons
(c) traditionally like by rich people only.
(d) show the warmth of the giver towards receiver.

PASSAGE - III
A man may usually be known by the books he reads, as well as by the company he
keeps; for there is a companionship of books as well as men; and one should always live in
the best company, whether it be of books or of men. A good book may be among the best of
fiends. It is the same today that it always was and it will never change. It is the most patient
and cheerful of companions. It does not turn its back upon us in times of adversity or distress.
It always receives us with the same kindness; amusing and interesting us in youth comforting
and consoling us in age
06. According to the writer. a man may usually be known by the books he reads because
(a) his reading habit shows that he is a scholar
(b) the books he reads affects his thinking and character
(c) books provide him a lot of knowledge
(d) his selection of books generally reveals his temperament and character.
07. Which of the following statements is not ture?
(a) Good books as well as good men always provide the finest company
(b) A good book never betrays us
(c) We have sometimes to be patient with a book as it may bore us
(d) A good book serves as a permanent friend.

ACE Academy
286

COMPREHENSION

08. The statement A good book may be among the best friends, in the middle of the
passage, means that
(a) there cannot be a better friend than good book
(b) books may be good friends, but not better than good men
(c) We have sometimes to be patient with a book as it may bore us
(d) A good book serves as a permanent friend
09. Which of the follwing is opposite in meaning to the word adversity occurring in the
passage?
(a) happiness
(b) prosperity
(c) progress
(d) misfortune
10. Which one of the following would be the most suitable title for the passage?
(a) Books show the readers character
(b) Books as mans abiding friends
(c) Books are useful in our youth
(d) The importance of books in old age
Keys for 1991:
01.b 02.c 03.b

04.a

05.b

06.d

07.c

08.d

09.b

10.b

También podría gustarte